Обсуждение:Теория относительности/Архив/1

Материал из Lurkmore

Перейти к: навигация, поиск

Содержание

Быстрее света (нубский вопрос)

я йентих ваших физиках я не силен, но праздное любопытство одолевает. Просветите, плз, неразумного. Вот допустим из одной точки, в противоположенных направлениях вылетели две неведомых ебанных хуйни и стали равномерно ускорятся. И доускорялись до 0,51с каждая. Соответственно, относительно друг-друга они должны двигаться со скоростью 1,02с и потерять всякий визуальный контакт, так ведь? И если да, то когда именно они потеряют друг-друга? когда скороть разлета превысит скорость света или они еще долго будут "видеть" друг-друга?

  • Нет. Скорость в ТО не аддитивна. Т.е. её нельзя просто складывать. Есть специальная формула для вычисления того какая скорость будет у одного объекта по сравнению с другим который летит относительно третьего (пересчет скоростей). И по этой формуле ебанные хуйни относительно друг друга никогда не будут лететь быстрее 1,000c (0,51c не просто складывается с 0,51c, а хитрожопо). И вообще, если рассмотреть этот опыт из неподвижной ИСО в которой он собственно сформулирован, то наглядно видно что свет полетит и долетит от одной хуйни до другой полюбасу. То же самое должно быть разумеется и в их ИСО. Если взять первый постулат и попытаться разлиновать листочек с бумажкой - как раз получится правило сложения этих скоростей по СТО.
    • Звучит круто, но я хуй чего понял (ну кроме того, что нехуй складывать скорости). С другой стороны я понял другую вещь, но понял ее скорее всего неправильно. Но даже с точки зрения нерелятивиских мозгоебств: гогда между двумя НЕХ расстояние становится 1 св.год, одна из них испускаеть луч любви в сторону другой. через год луч достигнет позиции где была вторая НЕХ, успевшая, к тому времени пролететь еще 0,51св.г. Ну а дальше старая как мир история ахиллеса и черепахи, но с хеппиендом. Это понимание хоть немного похоже на правду?
    • Алсо, я вроде осознал главную загвоздку в собсвенных размышлениях: свету похуй на то, что расстояние между объектами увеличивается быстрее чем его собсвенная скорость, ибо ему похуй, испускает ли его стоячий объект или летящий на невъебенной скорости, ему важна скорость объекта, который ему нужно догнать, а она не может быть больше чем у него, так?
      • С точки зрения Земли (ну или неподвижной ИСО), в которой левая НЕХ летит влево со скоростью 0.51с, а правая вправо - расстояние между объектами будет увеличиваться на 1.02с. Но это не скорость полёта объекта (или передачи информации)! С точки зрения одной из НЕХ (то есть, если мы представим её неподвижной относительно себя) скорость другого уже складываться с его не будет (точнее будет, но по Лоренцу, и получится меньше с). Типа того. И да, если левая НЕХ испустит свет, то он со скоростью 1с полетит за правой, а, так как та летит с 0.51с, то он её догонит.
  • Элементарно, Ватсон! Летит 1я НЕХ влево со скоростью 0,51с, вторая НЕХ - вправо со скоростью 0,51с. 1я НЕХ выпускает пук, и этот пук из точки своегопоявления начинает распространяться со скоростью 1с. Т.е. 2я НЕХ от него никуда не скроеца.

Про сложение скоростей

Вспомните пожалста, как меняется высота звука проходящего мимо вас автомобиля/поезда — примерно так: иииИИИИУУУуууу (кагбе эффект Доплера). Меняется частота звука, которую слышит наблюдатель. А скорость звука зависит только от материала, по которому этот звук передается, и не зависит от скорости самого источника. Скорость звука в среде (в воздухе) — это свойство исключительно самой среды, своего рода полоса пропускания канала, ширее которой ну ни как не пропихнуться. А также вспомните, что когда самолет прет быстрее скорости звука, то его в передней полусфере не слышно. Воздух не может передать эту волну быстрее, чем может.

Чел, окстись. Данное утверждение строго выводится в предположении, что вязкость равна нулю(а це не так http://window.edu.ru/window_catalog/pdf2txt?p_id=13978&p_page=7) лишь для газов. Для жидкостей и сжатых газов это мю уже нихуёво влияет самым подлым образом.Кроме того, зачем то придумали такую глупость, как "групповая скорость". У Релэя с Ландсбергом(коих я читал и могу судить) написан полный бред вроде "характерных точек" при рассмотрении биений 2х синусоид. И не ебёт их, что при любом модуляции синусоид три. Но три синусоиды брать низя, так как "характерные точки" разбегутся в стороны, и гениальное определение групповой скорости накроется медным тазом.

А кирпич, брошенный из этого же самолета будет иметь скорость равную сумме скорости руки с кирпичем и скорости самолета. Так вот людишки полагали, что и скорость света надо калькулировать идентично. Но пришел Эйнштейн, сказал «Вы — лохи. Скорость света есть свойство самой среды сей свет передающей. И не может наше пространство пропихнуть ЭМ волну быстрей 300 тыщ километров в секунду. Скорость источника ниибет.» —Tranz 21:49, 10 января 2010 (MSK)

  • Cool story bro. Но к чему ты всё это написал?
    • «Если вдруг не прав поправьте». Поправил. — К.О.
    Кхе, кхе. вот именно, что просранство не может передать ЭМВОЛНУ(!). а наше пространство из чего состоит, из атомов, которые по сути являются набором колеблющихся ЭМ частиц? а наше материальное взаимодействие основано на чем, на гравитационной природе? рассуждать об этих теориях еще рано, сыроваты они, требуют дополнительного анализа всех скопипизженных штейном формул.
    • Речь о том, что придрались к аналогии во втором абзаце цитаты. За зависимость скорости звука от системы отсчета Нобелевку, бегом!!!

Если кто не уловил тонкости вопроса: сложение скоростей по Галилею, использовавшееся в ньютоновой механике, пошло по пизде. Один и тот же лучик света, испущенный фонариком на поезде, будет двигаться с одной и тоже скоростью как относительно движущегося поезда, так и относительно неподвижного перрона. Зато частота света будет различаться. Но об этом — позже. То есть, для перрона он будет улетать со скоростью света. И для поезда — он тоже будет улетать ровно с той же скоростью света. И для фонарика. И для поезда. Одна и та же скорость. Даже если они двигаются относительно друг друга. Если тебя, мой юный друг, удивляет как такое может быть, то вспомни, что скорость звука тоже не зависит от скорости его источника. Будеш ты орать стоя на месте или мчась 1200 км/ч — скорости звука похуй. Зато для неподвижного наблюдателя частота звука будет различаться. Аналогия, да? Простите? Да ничего подобного! Скорости звука-то похуй, только звуковая волна будет двигаться с разной скоростью относительно неподвижного наблюдателя и едущего на поезде орущего. Имеет место то самое сложение скоростей из механики. (Если вдруг не прав поправьте.) Два раза не аналогия, ибо скорость звука зависит от выбранной системы отсчета, а в примере с поездом выше видно скорость света таким качеством не обладает (вообще то аналогия со звуком - это как раз самая что ни на есть эфирная модель, если что).

[[1]]
  • Этот ваш Эйнштейн, конечно, молодец, но о какой среде "сей свет передающей" он говорил, если эфир он отрицал?
  • Скорость волны звука как раз зависит от системы отсчета. Если вы летите вслед за волной - её скорость в вашей ИСО другая. Просто путаница с понятием скорости звука. Просто чаще всего этим словосочетанием сокращают "скорость звука в ИСО воздуха".
    А правда, что скорость звука не аддитивная величина?
    Ну если имеется ввиду что скорость звука не складывается со скоростью источника, то да, не складывается. С другой стороны складывается со скоростью приёмника (а более развёрнуто - складывается со скоростью воздуха).

К сожалению я не чукча, и чаще просто читаю. Но мне кажется что первое предложение: "Например для распространения акустической волны (то есть звука) нужен воздух" нужно подрихтовать до "хотя бы воздух" или как то так. Я пробовал, но нечитабельно получается или скатываюсь в унылый матан.

Оформление

Уважаемые, добавьте пожалуйста плашечку или же надо как-то красиво оформить начало статейки. что-то вроде: "Все в мире относительно" - чтоб читающие поняли смысл жизни. Salipot

Парадоксы

  • Народ, запилите в парадоксы про дуэль на тахионных пистолетах. Это круче чем шест и сарай, я гарантирую это.
    • Информацию по данной херне нарыл-напереводил, но уж больно скользкая его постановка, плюс к тому же сводится к известному временному парадоксу (чтобылобыеслибытыпереместилсявпрошлоеиубилсвоегоотца/деда/прадеда) и уже указанному в статье относительности одновременности. Кароче ненада ибо будет генератором срача. Tranz 01:04, 20 декабря 2009 (MSK)

С близнецами все ровно неувязочка. Как ТО может разграничить ИСО и неИСО. Допустим мы отправили обоих близнецов на ракете. Они разогнались. Все достигли скорости, теперь никакие перегрузки не действуют. Теперь мы одног7о отправляем на капсуле назад. Если забыть наше предыдущее ускорение, то это он ускоряется относительно ракеты соответственно он будет моложе. А если не забывать. То он теряет свой бонус молодости. Опят таки кто мешает двигаться не по орбите земли а прямо туда и обратно. Правильно ли я понял что рассинхронизация происходит во время ускорения? Получается что человек который разогнался и тут же остановился, будет так же молод как человек который разогнался пролетел 20 лет остановился?

Попробую объяснить на "палках" еще раз. Ускоренное движение во все времена и дали всегда было вещью интересной и действительно АБСОЛЮТНОЙ. В чём выражалась эта абсолютность? Давайте пока посмотрим на ускоряющуюся СО глазами Ньютоновской школьной физики. Самое первое что можно подметить - в такой СО перестают работать все 3 закона Ньютона. Т.е. падает, например, человек с небоскрёба и вот ведь какая оказия - он точно знает что суммарные действующие на небоскрёб силы равны нулю, но "о чудо" - небоскрёб ускоряется вверх (в системе отсчета связанной с падающим человеком) как будто к нему приложена титаническая сила. Более того, если взглянуть поширше вокруг мы обнаружим "удивительную" вещь - все тела во вселенной, вообще вся вселенная в такой СО приобретает (и вот тут важно) одинаковое ускорение противоположное тому, которое действует на человека и равное по величине. Ага... Вообще конечно всё логично - раз чел САМ ускоряется, то для него выглядит так будто это всё сущее вокруг ускоряется в другом направлении. Но тонкость в том что "всё сущее" этого ускорения не "чувствует". Тут всё логично - раз каждая часть это самого "сущего" ускоряется с одинаковой силой, то и никакого ускорения вселенского маштаба приборы принадлежащие ИСО вселенной в этом случае уловить не смогут. И вот тут всплывает маленькая тонкость - еще раз вспоминаем что в СО падающего человека на все тела во вселенной действует одинаковое УСКОРЕНИЕ, так, как будто бы некая сила, действует на каждое из них и тем сильнее эта сила чем больше масса тела, т.е. сила "подстраивается" по F=ma так, чтобы независимо от "m" обеспечить одинаковое "a" всем телам... и..... уже понятно наверное становится что такая сила у Ньютона уже была - именно так а не иначе действует ГРАВИТАЦИЯ, что собственно в статье тоже разжёвывалось. Итак, мы подошли в важному на сей момент выводу - с точки зрения физики Ньютона, чтобы удовлетворить выполнение 3-х законов Ньютона в ускоряющейся системе отсчёта надо "сделать вид", что в таких СО всю вселенную пронизывает однородное гравитационное поле, "сталкивающее" всю вселенную с одинаковым ускорением назад - и вуяля, мы можем в такой СО продолжать пользоваться тремя законами одноимённого чувачка и имеем полное соблюдение "одинаковости" физики и в такой СО и во всей вселенной - действительно акселероментр в глубоком космосе не будет показывать никаких ускорений в такой ускоренной СО, ибо все его части, грузики и ниточки ускоряются с одинаковым ускорением - следовательно это ускорение обнаружить не смогут. Вуяля! Однако конечно любой наблюдатель в такой СО прекрасно понимает что никакого реального гравиполя нет - оно фиктивно, и вот по этой фиктивности и выделяются ускоренные СО из всей чехарды и признаются "специальными, выделенными".

Ну вот мы и подобрались к ТО. А в ТО, как уже должно быть понятно, в силу преемственности принципов и азов, картинка получается ПОЛНОСТЬЮ АНАЛОГИЧНАЯ. Наблюдения из ускоренной СО описываются В ТОЧНОСТИ ТАК ЖЕ - введением во всю вселенную фиктивного гравиполя. Вот только в ТО гравиполе - это не сила, это ИСКРИВЛЕНИЕ ПРОСТРАНСТВА ВРЕМЕНИ. В силу последнего эффекты неИСО-вости (неинерциальных систем отсчёта) математически корректно и полно описываются в рамках ОТО. Если переложить эти эффекты в плоское пространство-время СТО, то можно прийти к примерно такой картинке: в СО на которую действуют ускорение время процессов впереди начинает ускорятся И ТЕМ БОЛЬШЕ, чем больше до события расстояние (прямо пропорционально), время позади - замедлятся и тем больше чем дальше расстояние до события (прямо пропорционально), вплоть до того что на некотором расстоянии (определяемом величиной ускорения) время процессов вообще останавливается (возникает подобие горизонта событий черной дыры). Можно рассмотреть такой пример. На некотором расстоянии от нас висит неподвижно палка. Мы начинаем ускорятся по направлению к ней и через какое-то время заканчиваем. Представим еще что на конце и в начале палки были часы изначально тикающие синхронно с нами. И вот что мы будем наблюдать: палка начинает ускорятся по направлению к нам, но этот процесс охватывает её "неравномерно" - задняя часть какбы начинает опрежать ближнюю - часы на ней начинают тикать быстрее чем ближние часы, да и сам конец ускоряется быстрее, как бы "проскальзывает во времени вперед", в итоге палка "сжимается" в соответствии с лоренцовым сокращением длин и когда мы прекратим ускорятся мы обнаружим что а) палка "сжата" по сокращению лоренца и летит к нам, б) часы на заднем конце показывают больше времени чем часы на переднем конце в) и те и другие часы показывают больше времени чем наши собственные, но теперь уже (когда ускорение прекратилось) тикают медленнее наших в соответсвии с преобразованиями лоренца. Вот такая картинка. Кто хочет - может повторить эти рассуждения для парадокса близнецов и для разных других комбинаций экспериментов якобы опровергающих СТО и убедится что картинка "склеивается".

Это стандартное объяснение парадокса довольно тяжело приложимо к подобным извращениям и разнообразным переворотам/ускорениям по той простой причине, что так сложнее, чем простое геометрическое объяснение через длины интервалов на диаграмме Минковского. "Если забыть наше предыдущее ускорение, то это он ускоряется относительно ракеты соответственно он будет моложе." - вот что это такое? Какое ускорение, в какой системе отсчёта, куда и насколько? То, что в ваших мыслях где-то витает и одному вам понятно, другим людям нужно объяснять подробнее, чтобы не было недопониманий, выливающихся в срач. Могу только сказать, что если один брат ускоряется, а другой нет, то ускоряющийся по-любому будет моложе.
Ускорение в СТО абсолютно. То есть, если кто-то ускоряется, значит это он ускоряется, и никто другой. В ОТО относительно даже ускорение (при условии введения фиктивных гравитационных полей).
Попробуйте лучше понять вот это:

http://upload.wikimedia.org/wikipedia/commons/c/ce/Twin_Paradox_Minkowski_Diagram.svg

фрики в истории правок

В истории правок был замечен некий ckotinko, который, с 99,9% вероятностью является одноименным жэжэшником. Чувак мегаохуенен: фоменковец, креационист, лунозаговорщик и, кажется, эфирщик. Тролль. С его творчеством можно ознакомится непосредственно в его же жежешечке. Буде внимательны.

Потрудитесь для начала разобраться в том, что сам защищаешь. Откуда взялась "неинвариантность" в классической физике например. Только не пиздя и разбрызгивая слюни, а на языке матана: мол так-то и так-то, и поэтому то-то и то-то. Можно начать с Э.Парселла, "Электричество и магнетизм", том 2. http://person.ipr.sci.am/vm/eLibrary/Berklievskiy_kurs_obshchey_fiziki_5_tomov/2_Parsell_Elektrichestvo_i_magnetizm.pdf , и, на основании учебника по матану объяснить, что за антинаучная хуйня написана на стр 245, после уравнения 29. Не проканают не ссылки на то, что учёные на опыте опровергли правила операций с производными, ни отсылка к авторитетам.
ммм.... я так понимаю - это ты упорно пичкаешь в статью истории про грустную судьбу полных производных в уравнениях максвелла? насколько я помню ты писал что в итоге получается баллистическая теория. тогда к тебе три вопроса: а) как быть с наблюдениями за двойными звездами, ведь по баллистической теории они бы могли весьма странно выглядеть? б) почему в ускорителях частицы не могут разогнать быстрее известной величины, ведь баллистическая теория этому не препятствует? в) во что превратится волна света в системе отсчета летящей всед за ней с такой же скоростью - какую энергию будет иметь и как колебаться?
БТР говорит(как и теория максвелла) о скорости на вылете, но не о её постоянстве. Не надо расширять поле действия теории по инерции на всю вселенную, У максвелла она считается как С*sqrt(n/p); далее, есть такая штука: fly-by anomaly. это скачок частоты источника(при с=const) или скачок скорости света(f=const) при подлёте к массивному телу(изза нее потеряли венеру-2 например, пару "рейнджеров", несколько марсианских зондов), её не объясняют ни СТО/ОТО, ни эфиродинамика. Т.е. разность хода будет лишь в пределах солнечной системы, если таковая обладает собственной fly-by anomaly. Эфир - всего лишь модель, как теплород, и при том - не лучшая. Теплорода нету, а рассчитаные на его основе паровозы работали, а потом оказалось что есть еще трение. Ну нету эфира, пусть будет поле, разницы-то ноль. На этом кстати эфирщики колятся со своим эфирным ветром. Ур.Максвелла же получаются вообще 4мя способами(эфир/алгебра клиффорда/векторный и ск. потенциалы и кто-то еще), у них ведь и продолжения есть (Хеннинг-Мейл, Ми, Борн-Инфельд). У Хеннинга электрон - это только волна а не частица, привет квантовой механике. В той же классической физике есть запаздывание потенциалов и там тоже нельзя разогнать что-либо быстрее скорости излучения. Причина в том, что ускоряющаяся частица черпает кинетическую энергию не от ктулху, а из ускоряющего поля, ослабляя его вокруг себя по з-ну сохранения энергии, и когда её скорость сблизится с скоростью поля, она просто его вокруг себя "проест". За скоростью света ускоряющее поле вообще начнёт тормозить частицу. Вообще СТО не плохая - она является хорошим преобразованием Лапласа с моей точки зрения: http://www.dialectical-physics.org/a01/11/ru/a0111ru.htm Плохо то, что она в религию превратилась. Пизданули у блавацкой(настольная книга эйнштейна кстати) 4х мерные пространства, возникновение мира из точки, и т.д. - бывает. Но на классическую механику бочку гонят необоснованно, и вообще говоря, жульнически.
вообще, я загнался конечно. нервы, алкоголизм, и всё такое, да. а тут еще креационистом обозвали. В физике не может быть "идеальных теорий". Это в матане мы четко знаем аксиомы и там работают в частности теоремы Геделя и т.д.. В физике мы их пытаемся угадать. Например, есть ли замедление времени в реале или это приборное замедление(ошибки работы прибора,которые мы не учли в теории). Но вот выкатили ТО и сказали - это - гениально и верно, кто против - враг народа. А между тем, если теорию нельзя опровергать "потому что это антинаучно", то эта теория ненаучна по критерию поппера и является доктриной.
Насчёт идеальных теорий - бог с ним. Тут всё понятно каждому кто хоть немного думал на эту тему. ТО не полна, это написано в самой статье. Ждём более великой теории.

Вопрос только в том - отменит ли новая теория ТО или дополнит. Я склоняюсь в сторону последнего. Вот опять таки - поэтому и вопросы. Вдруг в БТР что-то есть что отменяет ТО? Посмотрим ваши ответы. а) fly-by anomaly в качестве объяснения наблюдения за двойными звездами это я впервые встречаю. обычно в баллистике начинают говорить про межзвездный газ, который переизлучая свет даёт ему свою скорость. вообще сперва про ответ - ОТО, как раз заявляет, что каждое массивное тело "тянет" за собой шлейф пространства-времени с искривленными характеристиками - со всеми вытекающими - и свет там изменяет частоту и время прохождения пространства и радиоволны, и наблюдаемо это не только в размерах солнечной системы, но и отдельно взятой планеты. так что изменять частоту связи с зондом подлетающим к другой планете - по ОТО как раз придётся делать. Видимо так дело и обстоит со всякими венерианскими спутниками. Я знаю только одну fly-by anomaly о которой ученые трубят - это пионер. Вот там да, ничего непонятно. Но в любом случае использовать непонятный феномен в качестве объяснения БТР не объяснив самого этого феномена - это как минимум нелогично и ненаучно.

Так ведь тут такая ситуация, что сперва даётся феноменологическое объяснение(напр. закон графитации ньютона) потом теоретическое(обоснование исходя из чего либо). объективно - пионеры ведут себя точно также как зонд "галлилей"(его приводят как пример в педивикии). та же синусоида в колебаниях частоты. это хорошо ложится в ур-я максвелла. но тут у нас уравнение с 100 неизвестными. во первых есть вопросы к гравитации, есть мнение что поле гравитации конечно и оно невелико. те уж приливы описываются со словами "пусть по экватору прорыт кольцевой канал", а на глобусе я такого канала не видел. во вторых - к электродинамике - а насколько верны уравнения максвелла вообще? человечество их проверило лишь в малом диапазоне условий, микроуровня вообще никто в глаза не видел. Поэтому сперва описание "как есть", а потом объяснение. Самое глупое - это создать теорию всего и сразу. Лучше теплород открыть и чтоб паровозы поехали, а термодинамика подождет. Выбор теории в подобных науках - это больше личные предпочтения, ибо реальные законы неизвестны. (тут я покоцал ибо offtop)
Вашего мнения резко недостаточно для того чтобы вандалить статью домыслами. Об этом ниже. Если нечем подкрепить - лучше не трогайте статью.
Я, вообще-то, упомянул какова была предыстория появления СТО, какие были теории и т.п. А вот чего я не пойму - так это почему моё нормальное описание(человеческим языком, без мелкошпанюкоской фени) парадокса близнецов выкинули и вернули написанную вот ту ахинею, что сейчас там висит? Не ради флуда интересуюсь, мне за войну правок не платят, просто интересно, вот тот, кто эту хуету про алкаша васю писал, ему самому на фене объясняли СТО, или это какая персональная девиация психики?
И заметьте - я не устроил в статье срачъ на тему у кого теория длиннее. Хотите - снесу нахер все упоминания, жалко что ли? Мне - нет. Мне например не нравится текст после "Срачи по поводу этого опыта не утихают до сих пор", и я кстати не ебу, кто его вставил, хотя авторство моё. Но здесь не место - оффтоп.
Что касается газового объяснения, то оно, имхо, тоже ничего не объясняет - молекулы газа движутся с разными скоростями, так что если свет ими реально "тормозится", то получится что какой то фотон будет тормозится, какой-то разгонятся, третий приобретать перпендикулярную составляющую скорости - каша получится, а не изображение двойной звезды.
Вы чего-то фантастическое приписываете классической электродинамике, чего там никогда не было. В духе подмены производных. Фотоны какие-то... Можно бы было хотя бы того же Максвелла на первых 10 страницах прочитать, о какой модели идёт речь. Там МОДЕЛЬ - эфир. Такая гипотетическая непрерывная механическая среда. Там нет ни фотонов, ни "молекул газа". Это не значит, ни что эфир есть, ни что его нет, это просто способ вывода уравнений электродинамики из законов Ньютона. Т.е. если из А корректно вывели Б, то Б противоречить А не может. Всё.
касательно газов(и вообще всей "эфиро"-динамики) то в газодинамике есть большой баг - там при учете ветра складывают групповую и фазовую скорости, а так делать в общем случае нельзя. т.е. любой снос звука ветром не расчётный а следует из некоего умозрительного суждения. особенно это критично для поперечных волн - они медленнее, и в формулах для сноса поперечной волны надо писать хотя бы скорость не поперечной волны, а продольной. скорость волны - это колебания средних скоростей, а не сами скорости. то, что насчитал максвелл - это поперечная волна в несжимаемой жидкости(он из этого изначально исходил), а ней скорость продольных волн бесконечна - т.е. работает чистая БТР. Если можно сжать, БТР начинает врать. Если наложить турбулентность, то мы вообще считать её не умеем, за решение навье-стокса в 3д в США 1млн баков дают(забыл название инста). в openfoam есть 6 моделей на выбор, все жопные, а это боинг и орава американских НИИ. В МГУ вообще пиздец на лекциях доказывают - турбулентность в трубе при Re>1, суки хоть бы с опытом сверились(Re>2300). Т.е где-то систематическая ошибка, а где - не видят даже профессора. А вы хотите, чтоб после этого вам фотоны обсчитывали?
Гидродинамический эфир? Вай вай вай... Да вы хоть одну вещь приведите в пользу эфира вообще, гидродинамического или нет. Каким образом в баллистической теории вообще затесался эфир я вот не понимаю. Это ведь то же самое что сказать что скорость звука зависит от скорости источника. Что за эфир такой чудесный? "абсурдность неподвижности относительно газа" - это ваши слова в статье? Что за абсурдность? Разве трудно измерить давление с разных сторон и прикинуть покоимся мы относительно газа или движемся? Ничего трудно и абсурдного в этом нет. Или это уже не газ.
Месье, повторюсь еще раз: про "гидродинамический эфир" у Максвелла написано: http://vacuum-physics.com/Maxwell/maxwell_oplf.pdf ясно и понятно. Если хотите, можете Максвеллу доказывать, что он дурак, благо он вам уже не ответит. И скорость света там подсчитывается, и если у вас скорость света не складывается со скоростью источника то это только для абсолютно невязкого газа, а для него по тому же Максвеллу скорость света = бесконечности. Так что не-не-не, с механикой Ньютона всё в порядке, косяков там нету.

б) а вот идею что разогнать полем частицу быстрее скорости поля невозможно уже встречал. только вот весьма вразрез она идет с привычными представлениями физики о том что есть поле, а что есть изменение поля. скорость есть у изменения поля, а не у самого поля. поля статичны. если магнитное поле, которым в ускорителях разгоняют частицы, есть в какой то точке пространства - оно действует в точке пространства на заряд с конкретной силой, не зависящей от скорости частицы относительно поля. так же и в электростатике. если этот момент подвергать сомнению, то божич мой, вообще всю, вы понимаете, всю даже не электродинамику, а даже электростатику с её законом Кулона придётся пересматривать с самых основ. какие там уже уравнения Максвелла? там вообще что-то новое будет. Т.е. говорить об уравнениям Максвелла с полными производными и перечить при этом самое их основам - это тоже ненаучно.

Максвеллу я как раз не перечу. Полные производные - это как раз правильный учёт движений, и из него вовсе не следует независимость силы поля от скорости. Надо только векторы эл. поля погнуть ибо аберрация возникает, а синус там вполне себе "релятивистский". Касательно изменений, то во первых электродинамику еще Гаусс пересмотрел(запаздывание потенциалов). В части конфигураций(например полет заряда вдоль бесконечной заряженной плоскости или бесконечного провода) чисто релятивистский ответ выходит, с корнем. В смысле в СТО a=F/(m*гамма), а у гаусса a=(F/гамма)/m. При этом электродинамика Гаусса - это полевая физика, а Максвелловские ур-я - это эфиродинамика. Ну и в СТО понятно откуда он лезет. Формально три теории(гаусс,максвелл,СТО) дают одинаковые(местами) ответи, фактически - эти теории абсолютно друг от друга не зависят. Разница со СТО - калориметрически проверяема, и тут стоит такой пиздец, как опыт бертози(http://valveprincev.narod.ru/Bertozi/Part2.html). Т.е. нехорошо человек поступил, и такая проблема встречается довольно часто, что мешает. Все таки я физикой интересуюсь не чтоб "писали и коментили". К примеру, если положить условный "эфир" сжимаемым, как газ, то масса уже растёт, почти как в СТО(до 0.8с)(присоединённый условный "эфир" или поле), т.е. подобные опыты козлят всем гипотезам сразу. Может да, а может нет.
С теорией Гаусса не знаком. Статейку про Бертози почитал и заулыбался. Дело в том что она опровергает другого фрика, который тоже оперирует конечностью действия эл-маг поля и потому заявляет что предельная энергия электрона, разогнанного до "c" в поле в этом ИСО этого поля будет mc^2/2 (по вполне законному Ньютону). И ведь враль оказывается. А был так убедителен. Орал что калометрия даёт эти самые mc^2/2 и ZOG заставил всех умыться кто это исследует. Оказывается врал. Надо же. Как его утверждения в своё время трудно было проверить, так и эту статью про Бертози не могу.
Дело в том, что статейка про Бертози опровергает выводы Бертози, а не какого-то третьего лица, у которого "зог скрывает". Третьи лица могут утверждать что угодно, вплоть до того, что земля плоская, но его ебёт их мнение?

в) а вот на третий вопрос ответа не получил.

с точки зрения Максвелл(в варианте БТР) - для наблюдателя исчезнет электромагнитное поле, т.к. вклад волны в dA/dt обнулится, останется только падение амплитуды возмущений ~r^2. Т.е. приборно будет наблюдаться неподвижно удаляющиеся статические заряды источника в том виде, в котором их запечатлел свет с коим мы летим. По Гауссу свет останется(дальнодействие, увы) + будут наблюдаться продольные колебания эл. поля. Поправленный на близкодействие гаусс, КМК, сойдётся с Максвеллом с небольшой разницей - будет наблюдаться ненулевой заряд нейтрального источника.
Ага. А вот тут внимательно взглянем на статью. Кроме вас некому было написать там в параграфе где говорилось что эл-маг волна исчезает если лететь вслед за ней с её скоростью "что мол это только так в испорченном уравнении". Теперь оказывается вы сами говорите что она исчезает в этой самой "неиспорченной" БТР. Вот об этом я выше и говорил - вандалите статью, вандалите. Сами себе же уже противоречите. А раз так явно это уже, то попрошу не вандалить более. С Гауссом что-то поинтереснее получается - ненулевой заряд нейтрального источника - ЭТО КАК? Такое в природе обнаружено
Не обнаружено, наверно? Дык ведь у нас "в природе" и разогнать нейтральную частицу нечем до околосветовой скорости. Электромагнитная волна не исчезает, наблюдатель просто не может наблюсти эту волну как волну, т.к. производные по времени обнулились. А свет - это колебания поля, а не его постоянная составляющая. Берёте источник адской силы P=k*cos(w(t-x/с))/x2, и начинаете от него уёбывать со скоростью с. Волна никуда не исчезнет - просто вы будете видеть нечто другое: P=k*cos(w(t-ct/с)+φ)/x2=k*cos(w*0+φ)/x2=k*cos(φ)/x2. Источник - это колеблющийся заряд или заряды. По гауссу E зависит от второй производной по расстоянию - т.е. для двух движущихся в противофазе зарядов с сумме будет не ноль.

Постулаты теории относительности

Университет (педагогический, физмат) я закончил в 1999 году и поэтому прошу ответить уважаемых знатоков на вопрос из зала (в примечаниях): "Школьник Алексей Калистратов из Москвы интересуется, относительно чего эти (инерциальные, прим. - моё) системы отсчета движутся равномерно и прямолинейно?". Видимо школьник приверженец теории эфира и таким вопросом пытается подвергнуть сомнению постулаты СТО. По моему скромному предположению ИСО движутся равномерно-прямолинейно (или покоятся) относительно произвольно выбранной условной "нулевой" системы отсчёта. Для примера можно взять солнечную систему в качестве ИСО, а в качестве "нулевой" системы отсчёта можно рассматривать нашу галактику "Млечный путь" или галактику и Вселенную. В связи с этим ещё один вопрос, а наша Вселенная является ИСО? И если да, то по отношению к чему? Метавселенной?

Отвечаю на вопрос: инерциальные системы отсчёта, по определению, это системы отсчета, в которых выполняется закон инерции (1-ый закон Ньютона): тело на которое не действуют силы движется равномерно прямолинейно (или покоится). Опытным путём и умственными рассуждениями было выведено, что этот закон не нарушается в любой СО, если её связать с ЛЮБЫМ (любым, понимаете?) подобным телом, а можно и не связывать, если обеспечить чтобы система сама двигалась равномерно-прямолинейно относительно ЛЮБОГО подобного тела или ЛЮБОЙ другой ИСО. Как видите из самого определения - инерциальная система - система в которой выполняется закон инерции - совершенно похую относительно чего двигаться, пока это что-то само движется равномерно-прямолинейно. Понимаете? ПОХУЮ ОТНОСИТЕЛЬНО ЧЕГО, пока на это "что-то" не действуют силы. Таким образом, чтобы обеспечить истинную ИСО-вость, надо просто обеспечить отсутствие каких либо сил на какое-то пробное тело и двигаться равномерно-прямолинейно (или покоится) относительно него. Наша вселенная - сразу ИСО по определению, т.к. на неё со стороны просто нечему действовать силой. На отдельных участках же нашей вселенной приходится говорить о "локальных" ИСО - см. раздел статьи про ОТО.

Спасибо за ответ. Немного сбивчиво, но, кажется, смысл я уловил. Наверное я всё-таки лирик, интересующийся физикой.

Про математический аппарат

Слушайте, допилите кто-нибудь про критику математики в ТО, если это нужно, конечно. Таких статей много, но я не силен в математической софистике.--Кощей 11:18, 10 августа 2009 (MSD)
Формулу релятивиской массы надо убрать. Это понятие уже давно забраковано физиками. Сейчас когда говорят о массе говорят только о том что раньше называли массой покоя. Никаких других масс нет и впомине. Ни у фотонов, у которых вообще нет массы, ни покоя ни релятивиской, никакой вообще. И у массивных тел тоже есть только одна масса - m. Никаких m0 уже нет. "Релятивиская масса" - это вообще смешная штука, она разная в разных направлениях. Та формула, что приведена - она для продольного направления.

убрал, хотя формула забавно показывала, что до скорости света хрен разгонишь хоть что-нить, имеющее массу покоя
В таком случае можешь добавить формулу импульса, или лучше кинетической энергии, наглядно показывающую, что для того чтобы разогнать массивное тело до скорости света, нужно передать ему энергию, равную бесконечности. Как-то так :<math>T = \frac{m_0 c^2}{\sqrt{1- v^2/c^2 }} - m_0 c^2</math>
а может, латекс пофиксить? ну да, бесполезная наука, все дела.. ну зачем тогда нафик теги <math>?--Aikateri 15:08, 11 декабря 2009 (MSK)
Решительно двачую. Tenebrosus Scriptor 21:07, 12 декабря 2009 (MSK)

Парадокс шеста и сарая

Берем дрын длиной 10 метров и сарай с дверью длиной 5 метров. Разгоняем дрын до (почти) скорости света, при этом в соответствии с СТО его длина (с точки зрения наблюдателя, неподвижного относительно сарая) уменьшается, подберем скорость так чтоб это было в 10 раз - итого 1 метр. Дрын влетает в сарай, наблюдатель захлопывает дверь. Теперь то же самое с точки зрения наблюдателя верхом на этом дрыне - он сидит на дрыне в 10 метров, а на него надвигается сарай длиной 0.5 метра, ааааа!!!111, разумеется никак дверь не закроется. Вопрос - таки шо будет на самом деле? Срач гарантирован.

Хочу начать: очевидно же, что будет первая ситуация, так как в пространстве относительно Вселенной покоится именно наблюдатель (вернее не покоится, но скорость перемещения Земли с солнцем + Галактеко я не учитываю. Хрен с ней) с сараем, а не наблюдатель с дрыном. А раз так, то именно дрын будет укорочен и успешно уместится в сарай, а не наоборот.
так как в пространстве относительно Вселенной - в воздухе запахло абсолютным пространством. бггг.//ckotinko
Хуита, сокращение размеров будет вдоль направления скорости
Ну вот, про относительность одновременности уже написали в основной статье. (((--Кощей 11:42, 10 августа 2009 (MSD)
Главный вопрос шеста и сарая - пересечет ли задний конец шеста дверь сарая или нет (и почему). При этом стоит взять сарай с неразрушимыми стенками и одной дверью (через которую будет влетать абсолютно упругий дрын), а также 2х наблюдателей: одного у двери сарая и второго, сидящего на дрыне. Понятно, что не бывает абсолютно упругих шестов и сараев, и то, что все что будет происходить случится неодновременно тоже понятно; не понятно только ЧТО именно случится при такой формулировке, что увидят наблюдатели и почему.
практически то же самое, только событие "передний конец шеста вылетел из сарая" заменяется на "передний конец шеста ударился в стену". Ну то есть для того чтоп шест оказался внутри надо чтоп дверь (сразу после пролета заднего конца шеста) закрылась раньше этого удара. Если удар раньше двери то значит не влезло.
Тобиш как? Так влезло или нет (всмысле вообще есть такой момент, когда шест внутри полностью)?
1) Шест влетает в сарай. С точки зрения наблюдателя у двери он короткий, удара еще не было. Шест полностью влетает в сарай, его задний конец пересекает дверь, наблюдатель это видит; удара еще не было. Происходит удар. Абсолютно упругий шест останавливается, принимает свою реальную длину и его задний конец вылетает из двери во второй раз пересекая дверной проем (???)
2) Наблюдатель на шесте видит приближающийся короткий сарай. Он такой короткий, что шест в него явно не поместится. Когда передний конец шеста влетает в сарай и достигает задней стенки задний конец еще далеко от двери. Удар случается до того как задний конец пересекает дверной проем. Задний конец шеста так ни разу и не пересекает дверь.
Итого имеем: WTF?
Итого имеем не умеющих читать пидарасов. Шест останавливается блджад, о какой СТО вообще может идти речь?
А с точки зрения ОТО у тебя в момент удара произойдёт выделение ёб-ной кучи энергии (хорошо что пополам, ага), которую надо куда-то девать. Если по условиям задачи шест не кукожится, а также не излучает, то скорее всего он искривит пространство вокруг себя, так что дверь, таки да, всё время будет позади шеста. Не влетит. HAHAHA, DISREGARD THAT. В системе отсчёта шеста он тоже влетит в сарай полностью из-за скукоживания. Либо сам сократится (передний конец уже остановился, а задний ещё едет), либо пространство искривится и сарай растянется. В общем, нету парадокса.
Такая формулировка парадокса просто отсылает нас к другому свойству ТО - невозможности существования продленных в пространстве "абсолютно жестких тел". С точки зрения сарая весь шест в него поместится, дверь захлопнется и тут же УДАР о заднюю стенку, по стержню побежил волна деформации и он разорвется. С точки зрения шеста он передний его конец ударяется в заднюю стенку и по нему начинает бежать волна деформации. Но бежать быстрее скорости света она не может, поэтому задний конец "еще ничего не чувствует" и продолжает покоится. И "набегающий" с близкой к световой скорости сарай успевает долететь до этого заднего конца шеста раньше, чем прибежит волна деформации - дверь беспрепятственно захлопнется, т.к. у неё есть фора в преодолеваемом расстоянии по сравнению с волной деформации, которая распространяется от дальней стенки.
алсо, ботай
А вот напейшите в статье так, как тут, с наблюдателем на дрыне, а? Там далеко не сразу понял, в чем будет заключаться парадокс с точки зрения шеста.

Статья

Ящитаю статья годная. Если хоть один школоло прочтет ее и хоть что-то у него останется в мозгу, то уже афтар сделал доброе дело.

у школоты в моске ничего не останется, я это гарантирую. Ну то есть что-то там такое, про свет с паровоза, может и застрянет, но наружу оно вылезет только в классических формах бессмысленных и беспощадных срачей "а закроется ли ящик" (о, кстате, это обязательно надо допесать! это вам не сраный невзлетающий самолет, это эпичненько!).
Это ни разу не срач, этот случай элементарно объясняется разновременностью событий в разных СО.

В эту статью надо добавить как можно больше брейнфака.

Хорошо так, годненько. Не перегруженно как ЛМ-спиком, так и научными словами, хотя все равно ничего не понял) --MONOLIT 12:52, 9 августа 2009 (MSD)

Зато перегружено упоминаниями веществ.

Дюже двачую. Шура,пилите. Физтех-кун.

Матюков, блядь, до хуя. Филолог-сан.

Говно это, а не статья. Школота пытается обьяснить школоте то, чего сама не понимает. Поменьше читайте статью о Луговском и не пытайтесь строить из себя матанщиков, у вас это плохо получается. 646angel 13:54, 9 августа 2009 (MSD) +1

Што? По-моему, все очень доступно. Минимальный уровень понимания, достаточный для школоты и планктона, статья дает, ящитаю. LET THE FLAME WAR BEGIN!

Охуенно. На луркоморье в кои-то веки хорошая статья. Автор, если ты не боишься неймфажить, залогинься! Страна должна знать своих героев! --Insecticide 13:58, 9 августа 2009 (MSD)

статья действительно годная. Спасибо автору. Побольше бы таких статей... с физикоматановым уклоном.

Годная, полезная статья. Ваш ГСМ-кун.

/r/ более полное описание общей теории относительности. Интересно блин... Наверняка, тут есть физик-кун, который способен таки на пуговицах и табуретках рассказать что там и как. И, кстати, добавьте пару формул с зависимостью массы и времени от скорости (точнее от разности скоростей тела и света). Шоб было видно эпическое деление на ноль в пределе. Алсо неплохо бы добавить про временной парадокс. Сам не готов, ибо помню тока суть и могу где-нить по мелочи накосячить.

Неплохо было бы отобразить тот факт, что эта мозголомная теория является одной из основных (если не основной) дисциплиной на физико-матанных форумах. Ну и по поводу эквивалентности (а точнее неэквивалентности) массы и энергии...

Где тут можно поставить +1? :) Mad Fish 15:32, 9 августа 2009 (MSD)

достойная статья aziatkofag 15:39, 9 августа 2009 (MSD)

Я бы сказал что это омегаохуенная статья, да. --an Anonymous 15:49, 9 августа 2009 (MSD)

Таки статья годна. Для школоло. Для физик-кунов это слишком банально. Ваш физик-кун.

Не-не-не-не-не, анонимус, нет. Не надо делать из ТО очередную статью серии "К.О. спешит на помощь". Ибо К.О. тут вообще не уместен. Понять ТО можно только поняв матан, на котором она строится, любые объяснения "на пальцах" - это попытка черной тушью нарисовать радугу.

  • Ну, хотя бы СТО-то немножко понять можно. А вот ОТО без дифф. геометрии лучше и не пытаться.
    • Понять там без матана можно только то, что "мир чудесен и удивителен". То есть, если просто констатировать выводы, типа "скорость света - предел" и "близнец-космонавт моложе", тогда да. А вот как и почему, при чем тут скорость света, метрика Минковского, преобразования Лоренца и уравнения Маквелла - это без матана понять можно либо неправильно, либо случайно. Так что, я считаю, не надо наводить тень на плетень. В статье оставить постулаты и наиболее неожиданные и неведомые обывателю результаты ТО. Благо одних только наиболее известных "парадоксов" статьи на две-три хватит. Плюс срачи/опровержения/итд. А все попытки объяснить "умные слова на пальцах" либо выпилить КЕМ, либо заменить ссылками на педивикию.
      • Не согласен. При наличии знания предмета и фаниазии на табуретках и пуговицах можно объяснить что угодно. Плюс если тот, кому объясняют знает чуть-чуть матана, то процесс сильно упрощается. Например, гравитационное искривление пространства можно легко показать, перейдя на плоскость, которую искривлять в третьем изменении (че-то типа горизонтальной мембраны, на которой шарики от подшипников валяются и катаются). Если не получается объяснить на пуговицах - либо нехватает понимания (или фантазии) у объясняющего, либо слушатель ну совсем тупой. Так что просьба в силе. Еще бы кто объяснил нормально кто такой тензор...
        • Такие "объяснения", не подкрепленные пониманием матана со стороны интересующегося, могут сделать только хуже. Потому что аналогии, границы применимости которых физику ясны, для дилетанта превращаются в универсальные ответы. Ещё раз, проблема не в том, что конкретные моменты ТО нельзя объяснить "понаглядней". Проблема в том, что матан - это уже и так самый простой метод объяснить мир с точки зрения ТО, а аналогии могут служить только для придания фактам образности. "Объяснения на пальцах" плохи не тем, что из них нельзя верно понять ТО, а тем, что из них слишком просто не верно понять ТО.
          • Если тот, кому объясняют, не знает матана совсем и с абстрактным мышлением у него никак - то да. Но если аналогии правильные (подобрать их - тоже умение), а слушатель шарит в матане, но не настолько, шоб понять ОТО в виде формул с тензорами и прочей НЁХ, то результат будет положительный, ибо базовый матан препятствует "универсальным ответам".
          • Понять основные положения СТО вполне можно и без всякого матана. Хотя бы на уровне мысленных экспериментов, показать относительность одновременности, сокращение продольных размеров и замедление времени. А на картинках объяснить и геометрию Минковского, и как она к этому прикладывается. Понятно что практических знаний от этого не получишь, но хотя б какое-то понимание. Но в статье это будет выглядеть довольно странновато. Да и требует понятливого читателя.
            • это как на ноль делить. Вывих моска на таких качественных представлениях гарантирован. Формулы дело сложное, но железобетонное, а вот "больше-меньше, сжать-растянуть", учитывая приятные особенности ТО типа неодновременности событий, ведут в крейзу прямой дорогой.
              • Ну мы же здесь не физиков обучаем, в конце концов.
                • Зачем добавлять в статью "объяснения", если они могут только усугубить непонимание созданием ощущения ложного понимания?
                  • Подавляющее большинство людей и механику Ньютона-то понимают ложно. Но хоть какое-то представление они должны иметь, не так ли? (хотя, ТО от повседневной жизни довольно далеко отстоит)
                    • Для "хоть какого-то представления" есть википедия. Для "хоть какого-то представления" достаточно перечислить постулаты и выводы, без углубления в "как и почему".
          • Вы не правы. Матан - лишь язык. Язык функционален лишь в той мере, в которой им владеет носитель. Поэтому можно и русским/английски/французским языком излагать сложные вещи - будет длиннее, чем языком математики, но не невозможно! Беру с полки маленькую книжечку: Рёю Утияма "К чему пришла физика? От теории относительности к теории калибровочных полей" - эдакий покетбук на 200 страниц. Многое помню до сих пор - изложение "на пальцах". Рекомендую. Подобное же неоднократно проделывали наши учёные в библиотечке "Квант" и серии книг "Современная математика" - они ещё есть в букинистах или в сети. Не всё одинаково хорошо написано, но много простых объяснений сложных вещей.
        • К сожалению, только "при наличии знания предмета"! Ценю старания автора, но, с точки зрения физика, написанное в статье надо править от и до: изложено упрощённо-бытовое понимание предмета младшекурсником. Увы и ах, но времени особо нет корябать через прокси. Однако обещаюсь наведываться, чтобы задать неудобные вопросы для стимулирования дальнейших поисков.

Хорошая стотья. Порадовала. Спасибо Sotona 17:28, 10 августа 2009 (MSD)

Убрал сноски про монохроматическую волну. Все же это модель, а не реальность. --Кощей 00:14, 13 августа 2009 (MSD)

Нет, па-ада-аждите! Вы хотите сказать, что камень падает на Землю не потому, что "сила тяжести", как в школе учили, а потому, что "искривление пространства"? Вернее, что "сила тяжести" зто и есть "искривление пространства"?

  • Ага. ОТО относится к классу "геометрических" теорий, т.е. объясняет какие-то эффекты не силовым взаимодействием, а геометрией пространства-времени.
А-ахренеть! Т.е. реактивный двигатель или, скажем, рогатка суть "рукотворные целенаправленные искривители пространства-времени"? А практическое применение Единой теории поля - способ наипростейшего искривления для достижения великих целей?
      • Нет. Реактивный двигатель или рогатка - это "обычные" силы. Читайте внимательно раздел про ОТО - "странной" является только гравитация. Гравитация - это не сила (в отличие от электромагнетизма - той же рогатки или реактивного двигателя (да, да, эти вещи/силы - электромагнитной природы)), а искривление пространства времени. Я недостаточно точно выразился в прошлом комме - ОТО - это не просто геометрическая теория, а геометрическая теория именно гравитации. Что же касается ЕТП (Единой теории поля) - её основная неприкрытая цель - скрестить ОТО с КМ (квантовой механикой). Смотрите опять же концовочку раздела про ОТО. Эйнштейн думал, что если их скрестить - скрещивать больше ужей и ежей уже не придется, это типа последний уж и ёж - ОТО и КМ. Поэтому ЕТП и претендует на "теорию всего".

Может добавить приколов из Приключений мистера Томпкинса? Вроде там очень наглядные вещи. (для всех кто жалуется на сложность). Хотя, как, и сам не знаю. —Мимо проходил

Основываясь на цитате: «либо, как и всё «гениальное и простое», противоречащей опыту хуетой (РТГ, Лесажа).» убедительно прошу предоставить удобоваримую ссылку на ДОКАЗАННЫЙ ЭКСПЕРИМЕНТАЛЬНО фейл РТГ, статьи Зельдовича прошу не упоминать, в те годы принцип суперсимметрии был еще аполитичен (и на вопрос о причине искривления мировых линий в отсутствии переносчика, он отвечает "сильное колдунство" ). —Мимо проходил

Суровые сибирские лесорубы требуют картинок!

Альберт Эйнштейн смотрит на тебя со скоростью света
Проекция Германа Минковского на ущербное двумерное пространство

С каких пор lm стал науч-поп сайтом?

Соси хуй, планктон. Нерды тут всегда были.
Ух ты, какой агрессивный. Ты меня еще гуманитарием назови. И боговерчиком. Я не про нердов спрашивал. Но, помнится, когда двач еще был торт, а трава зеленой, на лм были статьи, объясняющие чем отличается RTFM от GTFO, а хикки от задрота, а не чем отличается бозон от фермиона, что и так делают 9000 "Техник Молодежи" и прочих "Вокруг Светов".
Были? да не менее половины админов сверхнерды.
Со дня основания.

Если уж на то пошло, то ТО весьма меметична. Чего стоит всем известное Е=mc2

Побольше бы таких статей! Тысячи интернетов, тебе, автор!

Блин, вот наконец-то хорошая статья, после кучи говн. Автору респект!

+1.
Респект авторам! Ахуенная работа- +100500. А тот замурзанный хуй, что просил картинки, пусть нарисует иллюстрации к статье,на букву "М"( я думаю она ему близка)- математику или множество.

Этой статье не хватает специальной олимпиады и торсионных полей

Без них статья не полноценна.

Пиздец, ёбаные пришельцы. ИДИТЕ НАХУЙ!

Хм... Здорово. Не думал что статья вызовет столько позитивных откликов. Раз так - обязательно добавлю раздел про самые известные парадоксы СТО и маленькую вводную "на табуретках и пуговицах" в ОТО. Разумеется без матана. Парадоксы, имхо, сами по себе достаточно лулзовы, как и заворот мозгов при введении в историю возникновения ОТО - в её постулаты, как они возникли. По идее и раздел про СТО тут - это не введение, а как бы краткая вводная в историю её возникновения. Превращать в хоть сколько нибудь подробное описание что и как происходит по формулам - неуместно, согласен. Поэтому даже "пуговицы и табуретки" очень поверхностные были и будут.

Добавь плиз поподробней про искривление пространства в ОТО.
Ещё ссылок на пару познавательных сайтов или срачей пихнуть.

Очаровательная статья. Я бы дала автору.

Я бы тоже, но пива.

Может moar убрать?

И так кошерно. А лучшее враг хорошего

Автор несомненно молодец, по-моему сносить можно обе плашки нахер.

По моей внутренней инерциальной системе отсчета статья - унылое говно и на хуй тут не нужна

и попробуйте, блядь, скажите, что я не прав :) хотя пару раз в разговоре с очередной тупой пиздой СТО очень помогала в качестве наглядного примера и объяснения того, что то, что верно для нее, может быть вовсе неверно для остальных. присоединяюсь к /r/еквесту парадоксов, дабы добавить в статью лулзов и интересности. в противном случае, снести КЕМ, так как 95% населения ни в хуй не впились все ваши теории. разве только чтобы посчитать насколько дней или недель среднестатистический россиянин, который ездит каждый день на работу в метро, автобусе или машине, удлиняет свою убогую жизнь по сравнению с деревенской дояркой

  • ты не прав
  • lm не ебут проблемы 95% населения, так что соси хуй, быдло.
а ты, уж прости, хоть и небыдло, но все одно - уебан. где ж ты, блядь, будешь, если 95% населения перестанут читать сраный лохомор? и да, я прав :)
95% о нем и так не знают.
Читать! Читать сука, а не править. Ты, и остальные 95%-1 можете выучить весь луркояз и разбавлять им свой гопосленг. Читайте хоть до усрачки, но не правьте. Алсо, да, продолжай сосать, не отвлекайся.
Опять же, lm не ебут проблемы 95% населения, хоть они читать перестанут, хоть сдохнут, одна хуйня.
извини, забыл подправить: "где ж ты, блядь, срать-то будешь?" и да, тебе не приходила в голову мысль о том, что если бы лохомору было действительно по хуй на 95% населения, то он был бы им недоступен? или ты считаешь, что все вокруг есть именно так, как об этом думает твоя головка? :)
(mailto:sage) Харощий история, брат.
извини, не силен в медицинских терминах

ОТО

Никто таки не сможет расписать чего-нибудь поподробнее про ОТО? Могу посоветовать выкурить: Мизнер, Торн, Уилер. "Гравитация", Бёрке У. "Пространство-время, геометрия, космология", Тейлор Е.Ф., Уилер Дж.А. "Физика пространства-времени".

Кстати, парадоксы-то описаны грамотно, но как-то уныловатенько...


Завтра про ОТО напишу вводную... Но в ней я почти полный нуб, кроме основных путей из которых она выводилась мало что знаю. Бррр... Риманова геометрия. Хуепиздец какой то... "Гравитацию" Мизнера и сотоварищей читал - с тем же успехом. Я, на самом деле, хотя и учился на физфаке, не физтех-кун с картинки, просто интересуюсь иногда темой.


И напишите кто-нить по-подробней про понятие горизонта событий не тока у чреной дыры, а в общем.

Парадоксы ОТО нужны? Я знаю один, но он в натуре парадокс и поэтому совсем не смешной.

Пойми друг, в этом мире нет ничего смишнее этого мира, поэтому давай, выкладывай что там у тебя.

ИМХО можно запилить и не смешной парадокс, но в раскрывающуюся ссылку, шоб просто так не отсвечивал. кому надо - посмотрят.

Благодраности и вся прилагающаяся хуйня

Спасибо, читал с удовольствием. В историю не лазал, но заочно чувствую пристутствия в статье RLHMа. Доставило просто фантастически.

Ты гонишь? Где ты почуял этого долбоеба? Может я просто не заметил в статье шизофазную стену текста? Статья хороша и как раз нашим сорокалетним девственником тут и не пахнет.
Все он верно учуял. Этот мудак уже начал вандалить статью.
Нет, не гоню. Честно говоря, мне положить на чье-то там отношение к кому-то, но при пересчете по пальцам рук людей продвигающих здесь мотан головных мозгов, он первым приходит на ум. Хотя да, сейчас уже вижу, что в статье мало лютой ненависти.

Статья действительно больше, чем просто отличная, общие принципы СТО и так_сказать, вступление у ОТО выложены аффтаром автором просто гениально. (Физик-кун, который со всёй этой хуитой работает)

+1 Статья супер, читал на одном дыхании. просто и доступно, авторам спасибо, побольше бы таких...жалко что в школе так физику не преподвали

Спасибо, Кэп! На этот раз я не зря зашел на луркмор и даже чуток подлечил свой рак.

История СТО

Что-то вы всё напутали. Эйнштейн ничего не выводил. Всё было выведено до него Лоренцом, Пуанкаре… и ещё какой-то чел был… Эйнштейн просто выкинул неуловимый эфир и переформулировал всё без него.

  • вики подсказывает, третий чел – Джордж Фицджеральд.
еще одно место подсказывает, что в методах выводов всех своих ТО у поциента наблюдается аналогия с Гейтсом, ничего нихрена не имея, тихонечко скоммуниздил чужие работы. когда работал в патентном бюро.

Какая школота поставила плашку с Петросяном?

Матан и СТО,

выносящие мозг,

Учите,

тупые дети!

Быдлу -

по заднице тысячей розг,

Автору -

сотни нефти!

Теория струн и квантовая механика

Реквестирую статью по теории струн

DO IT FAGGOT

Будет. --Кощей 09:47, 12 августа 2009 (MSD)

В теории струн обязательно указать на кучу срачей по поводу научности/псевдонаучноти теории, а также написать про многомерное пространство. Неплохо бы затронуть всякие браны и тд. Пару слов буквально.

В квантовой механике обязательно написать про кота шредингера, иначе статья будет неполной.

пилю потихоньку теорию струн --Tranz 17:26, 11 февраля 2010 (MSK)

А заодно и по квантовой механике. Больно интересная тема. Кроме того, крайне рекомендуется к просмотру фильм Элегантная вселенная в 3 частях.

Кагбе: теория струн, квантовая механика, кот Шредингера.

Опа, мой фэйл. Кота Шредингера не заметил.

КаГбЭ тогда надо писать и про лженауки (Кибернетика, Генетика), тем более, что "ан масс" чувствуется полная необразованность в этих вопросах, видимо, вследствие успешной борьбы с оными. Алсо, данную статью можно переименовать ближе к мему - "Всё относительно" и разбавить лулзами из лит-ры и анекдотов:

Великий русский писатель
Аристарх Геннадьевич Буроперов
сказал, что все относительно,
и был относительно прав.

Великий русский читатель
Владимир Исаакович Ланцберг
потому, вероятно, и великий,
что в этом абсолютно убежден.

Великий русский издатель
весь, какой был, уже вышел,
и эти великие мысли
вряд ли увидят свет.
(Ланцберг, 1988)
Моисей сказал, что всё от Бога,
Соломон сказал, что всё от ума,
Иисус сказал, что всё от сердца,
Маркс сказал, что всё от потребностей,
Фрейд сказал, что всё от секса,
Эйнштейн сказал, что всё относительно...
Сколько евреев - столько и мнений.
(анекдот)

Первая годная статья за этот год

И это прекрасно Мухомоар 10:47, 12 августа 2009 (MSD)


Хуйпиздазалупа

Вопрос: а почему за аксиому берется тот факт, что время(как временная ось) существует? Школьник-кун.

Потому что мы его наблюдаем, блджад. С пространством та же хуйня, представляешь?\
Мы наблюдаем только настоящее. Прошлое и несуществующее будущее есть только в голове у людей.
Мы наблюдаем непрерывный переход будущего в прошлое через настоящее. Алсо, всё твоё восприятие есть только у тебя в голове, но избави тебя б-г сомневаться в его реальности.
К.О., залогиньтесь. Я говорю о том, с какой стати можно "отматывать" время вперед-назад, изменяя состояние, положение всех объектов по вселенной, и о том, что понятие прошлого, точнее, устаревшая информация о некоторых объектах существует только в нашей памяти. Ладно, не буду позориться, необходимо разобраться в ОТО/СТО.
«Отматывать» время можно из-за строгого детерминизма событий, однозначности причинно-следственных связей как минимум в сторону прошлого (в будущее — нѣтъ, там пока только вероятности). Информация об этом самом прошлом хранится не только в нашей памяти (которая тоже весьма материальный объект), а вообще везде, вспоминаем про реликтовое излучение, например.
Реликтовое излучение здесь каким боком?
Таким, что это информация о событиях прошлого, существующая вне нашей памяти. Первое, что в голову пришло в качестве примера.
Таким образом, свет, который доходит до земли от галактик тоже является информацией о событиях прошлого, сществующей вне нашей памяти. Так что не реликтовое, а любое излучение. Реликтовое излучение - всего лишь очередной вид электромагнитного излучения, просто его источником выступают очень холодные тела.
Эээ... Вот только не надо про источники, реликтовое излучение было вполне себе горячее, только оно маленько остыло из-за расширения Вселенной.
Exactly. Только это «прошлое» в системах отсчёта тех самых галактик, для нас это самое что ни на есть настоящее, относительность одновременности as is. А вот реликтовое излучение «из прошлого» для всех систем отсчёта во всех галактиках, к нему не приебёшься :)
Кажись фкурил, хоть и с треском. Дает знать гуманитарное образование х_Х
А вот сабж. Хранить информацию о прошлом мы можем даже на 300мб винчестере из 1995 года, но это всего лишь, грубо говоря, "картинка". Каким боком идут различные путешествия во времени, старательно нарисованные в различных книгах/фильмах? При чем тут связывание пространства-времени? И как это доказывает, что временная ось существует?
Все «путешествия во времени» идут нахуй вместе с книгами/фильмами — за долбоёбские фантазии мы ответа не несём. Ещё раз для детей природы: существование времени наблюдается, а не доказывается.
Поняли, заткнулись.
Вопрос очень верный, но из другой области. Предлагаю покурить историю Людвига фон Больцмана (связь механики с термодинамикой) и статистической физики. Что же касается непрерывности пространства (квантЫ строятся опять же на основе этого допущения) - думаю, со временем наука дорастёт и до проверки таких альтернативных гипотез.
Из остаточных знаний, физика-кун, время нельзя отматывать в квантах, в ОТО можно - Эйнштейн разрешил!. Пруф из педигрипала Обратимость времени

А реален ли вообще мир?

Нет, конечно. Только что с того?

Мемично? Смешно?

Я прилёг на пол, чтобы вдоволь насладиться безудержным моим смехом над этой занятной юмореской.

Вопросы к Галилею

Это, тут у школоты появились вопросы к товарищу Галилею. Типа в гравитационном поле все тела двигаются одинаково. Однако злобная учительница в пятом классе говорила, что сила гравитационного взаимдействия F=G*m1*m2/r^2, а так как сила и вызывает ускорение, то два тела с разными m1 будут двигаться с разными скоростями в поле m2. Чзх?

  • сила действительно вызывает ускорение. а по какому закону? Вспомним F=ma. Значит a=F/m. Подставляя вместо F выражение для силы тяжести: G*m1*m2/r^2 имеем a=G*m1*m2/(r^2*m1), m1 сокращается и получается что ускорение не зависит от m1.
    • С этим кстати связан еще один срач: в G*m1*m2/r^2 - m - гравитационная масса, а в F = ma - инерционная. И соответствие одной массы другой чисто экспериментальное.
      • см аномалии бугэ. нет там никакого соответствия.
  • школота, ну нельзя же так фейлить.

Большего количества ошибок в СТО не видел

Собстно, вот. Писала какая-то школота-небыдло, возомнившая себя физиком. Покури учебник, блядь, для начала.

  • Не кормить!

Гуманитарии вопрошают

С искривлением пространства в ОТО относительно(!) понятно,хотя и приходится садиться за матан. но юный и неокрепший мозг не понимает : если ложкисилы нет,то почему камень,отпущенный свободно из руки,движется в одном и том же направлений?

  • Поскольку его мировая линия искривлена гравитирующей массой в нужном направлении. Tenebrosus Scriptor 03:12, 16 ноября 2009 (MSK)

Парадокс близнецов

Лучше бы автор написал, что сколько не выёбывайся, летая туда-сюда, а природу не наебёшь. Состарятся одинаково.

Это что за высер: Правка Анонимуса? Аффтар хоть сам понял, что имел ввиду? О каком "шлейфе" и "сверхсветовом движении" идет речь? Если этот бессвязный "пердеж" и правда сожержит хоть какую-то крупицу "логики", его нужно срочно перепилить, иначе - нахуй!

это в вашей любимой ТО есть фишка, что можно путешествовать во времени, если двигаться со сверхсветовой скоростью.
А это часом не экстраполяция "замедления времени при приближении скорости движения ИСО к скорости света"? То есть, если начертить график отношения "замедления времени" к скорости ИСО, и "продлить" его за 300,000, то получится, что время потечёт в обратную сторону => можно вернуться в прошлое (хотя опять же, двигаться с сверхсветовой скоростью низзя,и т.д., и т.п.).
Нет, это следствие из преобразований (Лоренца) координат событий при смене ИСО. То есть, если с одного космического корабля быстрее света послать сигнал в другой космический корабль, имеющий скорость относительно первого (досветовую), а оттуда обратно (по прибытии сигнала), то можно так подобрать скорости, что сигнал от второго корабля попадёт к первому до того, как первый пошлёт свой.

Для особо одаренных поясняю, что стареют вместе одновременно, а если проще пояснить - под временем мы понимаем все-таки "искривление" пространства, т.к ничего в состоянии покоя быть не может. хоть в какую глухомань вселенной ни отправляй со всякими световыми или сверхсветовыми скоростями. Чтоб утверждать парадокс - необходимо опеределить что вы имеете под величинами "время", "скорость", "масса" и т.д. для ВАШЕЙ ТО, а не брать эти величины с определениями и свойствами применительными для околоземного мира. И еще ответьте тогда: ближайшая к нам звезда альфа Центавра находится на расстоянии 5 световых лет. получается, что свет испущенный звездой (по нашему летоисчислению) в 2005г и зафиксированный нами в 2010г отличается от испускаемого уже в данный момент времени (2010год) на туеву хучу лет? как тут применить парадокс близнецов? т.е. свет нихуя не 5летней давности до нас дошел (по нашим подсчетам)?

суть путешествия в прошлое тахионов в преобразованиях Лоренца. Точка. Никакие конечные скорости сигнала и прочая хуйня здесь вообще ни при чём. Время - ни хуя не искривление пространства. В СТО пространство-время плоское, просто с хитрожопой псевдоевклидовой геометрией. Одновременность в СТО понятие лишнее, и обкуренные физики, которые достигли 4-мерного мышления вообще им никогда не пользуются, потому что оно нужно разве только в школьном курсе СТО, где на пальцах показываются эффекты замедления времени, сокращения расстояний и относительности одновременности (причём первые 2 являются следствием 3-го). По-моему уже довольно перетирать объяснения СТО на школьном уровне и пытаться подстановкой цифр притянуть за жопу какие-то неразрешимые парадоксы, которых в СТО нет.
скорее всего, из-за этих самых парадоксов и несостоятельна ТО. так что смысл - мы вернемся в прошлое, что все вращается вокруг земли и т.д. и таким образом, нахуй тогда смотреть на звезды, если они хуй знает о чем нам мигают относительно себя. а пытаться рассказывать про парадоксы и обсуждать, вычислять их - эт лишь засирать мозги разговорами ни о чем.
ай, скока пафоса. "школьное", "на школьном", "притянуть за жопу". За жопу притянуты как раз таки объяснения того же парадокса близнецов, который, ви таки будете смеяться, неразрешим. Причем в самой статье жуется жвачка про ускорение, придуманная ещё Паули, кажется. А ведь объяснение уже лет 50 как само по себе "устарело" - пусть автор вспомнит про третьего близнеца того же, если он, кроме курса Франкфурта и Ландау-Лившица что-нить читал-слышал. Кстати, статью как будто википидор писал, спецом посмотрел - похожая слоупочная хрень, через сигналы "разрешается", плюс просто, ускорение - не ИСО - проблемы негров шерифа не ебут. Впрочем, релятивиста даже могила не исправит, спасибо, хоть плевок в Максвелла убрали из текста. Даже что-то обсуждать трудно, просто убивать себя об стену релятивистской тупости и отсутствия даже и математической опрятности. С другой стороны, что с бедных людей взять, если они между собой договориться не могут... Нашёлся б честный релятивист, подсчитал бы количество вариантов СТО, трактовки в них явлений и рисований "пары формул" изменения длины, времени, доплер-эффекта, например. Скока набралось бы, 10, 20, 100, больше? И эти люди ещё набираются наглости приписывать себе какие-то практические доказательства... Хотя говорят как-то неубедительно, всё тише эти звуки, что радует -:).
Ви таки будете смеяться, но парадокс этот даже парадоксом сложно назвать (потому как ничего противоречивого в нём нет), при грамотной постановке (почитай Бёрке, например) отсутствие противоречий очевидно. Если же пытаться внести сюда "замедления" времени и прочую хрень (которые возникают-то только из-за введённого понятия одновременности, которое, вообще говоря, лишнее), то можно попытаться и чего-нибудь запутать, типичная тактика демагогов.
Короче. Я не шибко силен в физике, но ёбаная Настя, по логике вещей два близнеца не разойдутся в возрасте. Ведь до назначенной звезды (расстояние от Земли = 1 световой год) одному из них (со скоростью света) лететь целый год в одну сторону! Вот если бы близнец летел хотя бы в два раза быстрее, то обернулся бы за год (на Земле прошло бы два). А если бы в четыре..
Юра мгновенно разгоняется почти до световой. Для него пройдут считанные дни-минуты (зависит от скорости). А "лететь целый год" - это он для землян будет.

Эт все на два поста выше, тонкий троллинг что-ли такой? 0_0

Я в физике не силён

...но даже мне стало кое-что понятно. В частности, откуда растут ноги у идеи о гравитации как искривлении пространства-времени. /r/ сотни нефти автору и плашку notcrap в статью!

вынос тела начинающегося срача

Нельзя ли рассмотреть случай, когда время полета намного больше времени ускорения, допустим, если dT_accelerate — разница за счет ускорения-торможения (у любого участника), то общее время полета T_flight >> dT_accelerate? Also, в случае замкнутой модели Вселенной свет, однажды вышедший из одной точки может вернуться в ту же точку «с другой стороны». В этом случае фаза разворота вообще отсутствует. С уважением, физик-экспериментатор.

Вообще-то, всё ещё хуже: разгон/торможение — разовый процесс, он даст фиксированный сдвиг по времени. А , и сдвиг от полёта может быть разным. С уважением, физик-теоретик.
Не понимаю, какой смысл баловаться с циферками и чё-то пытаться там накручивать? Нарисовал диаграмму, прочертил кривую и прямую и убедился, что всё работает.
Кому баловаться, а кому считать. Если подсчитать, то выходит, что "гравитацыя", порождённая ускоренной СО, действует тем сильнее, чем дальше находится объект на который она действут. Поэтому считать не надо. Надо начертить типовую картинку по методичке, произнёсти пару умных фраз типа "вот видите, наука всё объясняет" и сделать вид, что всё объяснено. картинку можно не понимать, надо уметь чертить и знать фамилию минковского
Вы бредите. Ускоренная СО никакой "гравитацыи" не порождает, хотя имеется локальная математическая эквивалентность двух явлений. Локальная, а не ослабление/увеличение с расстоянием. А что вы хотели сказать пафосной речью про типовую картинку? Разве надо выдумывать какой-то оригинальный способ доказательства того, что уже множество раз передоказано?
локальная, математическая. вам русским языком сказали, какая проблема в "объяснении" парадокса близнецов заложена: эксперимент всегда можно разбить на часть с ускорениями(разгон/торможение — разовый процесс) и без ускорений(лететь с большой скоростью можно сколько угодно). Но считать, как вы сами сказали, вы смысла не видите. То ли дело картинку нарисовать и сказать что тут всё уже давно доказано.//ckotinko
Дело в том, что эти части - с ускорением и без ускорений - взаимозависимы, и как бы вы ни подбирали расстояния, числа всегда сойдутся. Если вы увеличите часть без ускорений, то часть с ускорением будет на более далёком расстоянии и её влияние, соответственно, будет сильнее. Это можно воочию увидеть на диаграмме Минковского: две мировые линии полёта в ПВ, начинаются в одной точке и кончаются в другой точке, "длины" (интервальные) этих линий - время часов. Какая линия длиннее (по псевдоевклидовой метрике), тот близнец и старше. А все одновременности на графике прочерчиваются косыми линиями под углом, зависящим от мгновенной скорости. Проблема только в том, что диаграммы Минковского в научно-популярной литературе не разъясняются, хотя на них отсутствие каких-либо проблем очевидно тем, кому не очевидно аналитически. Это всё уже почти сотню лет назад было изучено. Займитесь уже самообучением.
ад и израиль. Говоришь челу: "была проблема Х. вы ёё решили, введя Y. но из Y либо по прежнему не закрывает проблему X, либо из него следует другая проблема Z". он тебе в ответ: "проблемы Х не было. мы ввели Y. из него следует фича Z. так нам завещали сотню лет назад оццы-основатели". имеет ли смысл продолжать разговор с человеком, не способным прочитать написанное?//ckotinko
Не нравится? Жуй. Проблемы никакой нет (предъяви расчёты с проблемой, покажи, как у тебя интервалы (которые по преобразованиям Лоренца инвариантны) с разных точек зрения оказываются разными, а точнее: интервалы жизни одного Близнеца с точки зрения самого Близнеца и его же с точки зрения другого Близнеца), сам эффект близнецов можно объяснить разными способами: от школотовского до вполне серьёзного. Можно и с точки зрения ОТО. Имеет ли смысл продолжать разговор с человеком не пытающимся разобраться в теме?
Такое впечатление, будто с ботом переписываюсь. Вот есть у него в методичка "покажи интервалы", он и выдает это. Шаг в сторону от методички - и возникает неспособность понимать написанное. Причем вы не одиноки: тут ниже еще один идиот оказался не способен осилить определение слова "парадокс".//ckotinko
Пилять. "dT_accelerate — разница за счет ускорения-торможения (у любого участника), то общее время полета T_flight >> dT_accelerate" "разгон/торможение — разовый процесс, он даст фиксированный сдвиг по времени. А , и сдвиг от полёта может быть разным". Сдвиг от полёта, естественно, может быть разным, но от длины полёта автоматически зависит место выполнения разворота и эффект от "ускорения-торможения". Обе эти величины скоррелированы и все проблемы нахуй исчезают. Это во-первых. Во-вторых, нормальное рассмотрение (как я уже много раз пытался объяснить) предполагает рассмотрение с точки зрения пространства Минкосвкого, его псевдоевклидовой геометрии и ненавистным вам интервалов. А случай замкнутой Вселенной пускай ОТО рассматривает.
  • ниачом


Also, в случае замкнутой модели Вселенной свет, однажды вышедший из одной точки может вернуться в ту же точку «с другой стороны». 

Вселенная как бы расширяется. Причем так резво, что даже свет не успеет пролететь полный круг.

E=mc2

Кто сказал, что принцип действия ядерных бомб исходит из этой формулы? Это что вообще за хуйня? То есть через нее конечно можно рассчитать энергию, выделяющуюся при распаде ядра. Однако принципиально ядерная энергия берется из-за разрыва энергии связи. Точнее, если конкретно взять например уран-236 (образуется из урана-235 поглощением нейтрона), то эта энергия берется из кулоновского взаимодействия протонов. То бишь когда разрываются сильное взаимодействие, связывающее эти самые протоны, их начинает ускорять кулоновская сила, и при этом разумеется оные начинают излучать гамма-лучи. Остается пара-тройка лишних нейтронов, которые превращают последующие ядра урана-235 в уран-236.

Имхо, если "можно рассчитать", то это уже говорит о том что формула имеет самое непосредственное отношение к происходящему. Дефект массы то есть? Есть. Откуда он взялся? Если я всё правильно понял, то как раз, когда ядро урана образовывалось требовалась масса энергии против кулоновских сил, чтобы их "упаковать". Это энергия выразилась потом в дефекте масс ядра урана как раз по обозначенной формуле. При распаде она высвобождается, да в виде кулоновской энергии - но ведь и формула то про взаимосвязь энергии и массы. Какая именно энергия - не уточняется, ибо всякая. В данном случае работа против кулоновских сил. В термояде гамма-квант выделяется - тоже вид энергии.
Дефект масс вообще здесь не причем. Он появляется не только из-за энергии связи, но и из-за того, что массы протона и нейтрона не равны одной атомной единице массы. Эта величина вообще не имеет физического смысла. Ее в химии чаще используют, а не в физике. А с энергией связи такая проблема заключается в том, что если мы будем считать массу этой самой энергии, то внезапно она у вас получится отрицательной. А это бред чистой воды. Отрицательной она получается из формулы для энергии связи M(A,Z)c2=(N*mn+Z*mp)*c2-W(A,Z), где W(A,Z) - энергия связи, а остальное думаю понятно. И чтобы реакция была экзотермической (то бишь, чтобы выделялась энергия), необходимо, чтобы энергия связи возрастала для продуктов, получившихся в реакции.

Но с другой стороны в статье проговаривается что ТО использовалась при создании ядерной бомбы - вот это неверно.

Это верно - об этом даже Курчатов Сталину говорил (луркайте биографию К.).

По этой теме в статье написана хуйня. Из раздела "не знал, да забыл", за такое на экзаменах ставят 2.
При ближайшем рассмотрении все оказалось не так плохо. Но! Кто так строит, кто так строит. Специально писалось, чтобы еще больше запутать? :)

Школьная физика

Умнее говоря, ИСО — это системы отсчета, в которых выполняется первый закон Ньютона.

Вообще-то 1 закон Ньютона формулируется до смешного просто: "Существуют инерциальные системы отсчета" => кто-то пытался выпендриться, но зафейлил.--62.141.118.146 18:04, 2 марта 2010 (MSK)

Трололо

Если всё относительно, то значит, что и относительность относительна => абсолютное существует

  • А вот хуй там, не всё относительно.
  • Значит, есть что-то не относительное. Но, поскольку относительно всё, то не относительного быть не может.
  • А поцчему вы решили, что всё относительно?
  • Потому что, как ни странно, всё относительно. А то получится, что ТО страдает ВП
  • Да ну! ТО никогда не говорила, что всё относительно. Те же интервалы в СТО абсолютны.

А есть ли парадокс подводной лодки?

Если предположить, что гравитация и свет идентичны и являются излучением разного порядка (квантового и атомного соотв.), то мы имеем следующую картину: гравитация это такой квантовый свет.

Таким образом, любая песчинка - светится "гравитацией", равно как любая гигантская песчинка начинает испускать свет.

кванты придумал планк, поскольку ничего кроме распределения Релея и закона Вина не знал. посчитать руками, как это сделал Кельвин, он тоже не смог. Пришлось "постулировать". а все парадоксы у вас от погнутых хэвисайдом производных. и вообще, вы погуглите на тему "электромагнитная масса".
Вот не надо наезжать, парадоксы и аномалии не у меня, я не матанщик. И если бы "электромагнитная масса" была объяснена как Настоящая статья - с лёгкостью бы, а так - я нихуя не понял чем это отличается и в чём профит/фейл. ГСМ же!

При движении со значительной скоростью уровень "светимости" для внешнего наблюдателя падает т.к. время затраченное на испускание света "сократилось", а время на получение "увеличилось". Итог: пропорциональный рост/падение веса компенсируется обратнопропорциональным падением/ростом гравитации. Сорри за ГСМ формулу не напишу.

Таким образом, и человек падающий в лифте рядом с яблоком, и подводная лодка движущаяся со скоростью света хотя и "набирают" "вес" (а следовательно и "гравитационную мощь"), но и значительно "снижают" свою "светимость" и, соответственно, компенсируют разницу взаимодействия. Иными словами: несмотря на то, что из-за движения вес меняется, итоговое гравитационное воздействие остаётся неизменным вне зависимости от скорости движения и массы объекта.

Поправка: возможно, правда, объяснение того что и песчинка и газовый сверхгигант упадут на горизонт событий примерно одновременно из-за того, что газовый гигант тупо "ярче" и лучше делит на бесконечность "свет" чёрной дыры, но это надо думать, а мне лень.

алсо, если кому интересно есть и такая моя же ахинея:
1) Причина провала эксперимента с "Гравитационным ветром".
Тупо и банально: не учтено то, что и Солнце, и Земля создают свой ток гравитационного ветра и потому замер не имеет смысла, так как проводится, фактически, в "стоячей воде".
Зато объясняются отклонения в движении спутников - их смывает. И потому же, кстати, электрон не падает на ядро: парит аки дельтаплан.
Повторяю для инквизиторов-матанщиков: вокруг каждого объекта обладающего гравитационным моментом есть своё гравитационное поле, которое под внешним воздействием может изменять форму и формировать "течения" (зоны меньшего напряжения с постоянным направлением вращения). Любой объект попадающий в эту область изменяет своё направление так, чтобы наименьшим образом сопротивляться энергии поля, однако при высокой массе и скорости (планеты) подобное воздействие минимально и потому кометы падают на Землю.
2) Гравитация есть w:электромагнитное излучение идущее после т.н. гамма-излучения. Потому оно и не действует на электромагнитное поле: частоты разные. (см. w:Бритва Оккама)
Возможное следствие: w:гамма-излучение наносит урон именно предгравитационным воздействием на атомы. Воздействие на электроны (согласно данным вики) может быть объяснено волновым гравитационным воздействием в том числе.
"Если не подходит простое решение - значит надо искать ещё более простое!"(с)Ithappens.

Лурк = образовательный!

Спасибо авторам статьи! Я - бухгалтер(-кун), и нам эту вашу физику преподавали, не вдаваясь в подробности. Я искренне пытался понять, что к чему в сабже, а главное - зачем так??7 Общеизвестно, что 3+2=5, 300+200=500, а 300 миллионов + 200 миллионов = 500 миллионов! Но если, например, корабль летит со скоростью 200тыс.км./сек., а ему навстречу летит частица со скоростью 300тыс.км./сек.(скорость света), то ПОЧЕМУ скорость их сбляжения будет равна скорости света, как будто бы корабль стоял на месте?77 И, заметьте, я не спрашивал себя - как, я спрашивал - почему так, а не иначе? Я долго искал ответ. Читал и Википедию, и изучал специальные сайты, но не понял нихрена! И, наконец, наткнулся на эту статью, в которой красиво и в доступной мне форме даются ответы на все мои вопросы, за что сугубо вас и благодарю!

Абсолютность одновременности

Я сам не физик, но мой дед - физик-ядерщик с ФГМ в терминальной стадии, всю жизнь испытывавший попаболь от СТО и тем боле ОТО Эйнштейна и всю жизнь пытавшийся ее опровергнуть. В основном все его попытки были неудачны, но он подступался с разных сторон и нашел верный путь: он придумал эксперимент, который опровергает постулат относительности одновременности. И, как смог, объяснил мне его. Известно, что Эйнштейн в качестве аргумента ОО придумал эксперимент с передачей электромагнитного сигнала от одного будильника к другому, заявив, что это единственный способ синхронизации систем отсчета.

Последняя фраза - бред. Не было такого у Эйнштейна. Синхронизировать светом он конечно синхронизировал, но ничего такого что это единственный способ - нет и не было. А светом это дело делать сугубо удобно для вывода дальнейших формул из второго постулата, вот и все дела.
но ничего такого что это единственный способ - нет и не было. ололо. дики и неграмотны релятивисты, преобразований, сохраняющих инвариантным волновое уравнение, аж 2 невъебенных класса. Причем, плачьте суки - они все эквивалентно описывают все релятивистские эффекты СТО, за одной маленькой разницей: это как раз способ синхронизации.
Ой да ладно вам, какая разница - сколько этих преобразований?
этих преобразований - как у квантунов параллельных вселенных в теории множественных миров. если они все "работают", значит - они все не работают, а являются отображениями в фиктивное пространство. при помощи отображений, например, на ОЦОСах цепи рассчитывают.
Они дают результаты, которые подтверждаются экспериментами. Что ещё можно подразумевать под "работают"?
А ещё придумали эксперименты подтверждающие/опровергающие наличие эфира (на отсутствии которого базируется СТО Эйнштейна). Но физики их упорно игнорируют.

Эксперимент деда выглядит следующим образом: будильники синхронизируются не с помощью сигнала, идущего от одного будильника к другому, а через механический импульс. Выглядит это так. Есть прямая а. На ней стоит ряд будильников. Есть прямая б, она пересекает прямую а под углом не равным 0 и не равным 90 градусов, точка пересечения находит в стороне от ряда будильников. Прямая б движется равномерно параллельно или перпендикулярно прямой б с тем чтобы Если прямая б пересекает точку, в которой находится будильник - он настраивается на определенное время. Расстояние между будильниками известно (одинаково). Скорость прямой б известна. Вычисляем скорость движения точки пересечения прямых, вычисляем время движения от одного будильника к другому, запускаем прямую б -вуаля, будильники настраиваются на одно и тоже время! Логику этого эксперимента дед назвал принципом гильотины. Вообще мне оба эксперимента кажутся дебильными, поэтому я выскажу лично свое мнение, опять же на примере эксперимента. Есть три системы отсчета: лампочка и два будильника. Оба будильника находятся на равном расстоянии от лампочки. По достижении фотонов света каждый будильник настраивается на определенное время. Поскольку фотоны будут преодолевать одинаковое расстояние и скорость их одинакова, будильники будут настроены друг на друга одновременно (и не надо даже вычислять разницу, потому что ее нет). Ура, торжество механики: S=v*t. Одновременность абсолютна, потому что две системы отсчета можно синхронизировать с помощью третей.

>он придумал эксперимент, который опровергает постулат относительности одновременности
Причём мысленный. Значит, раз, относительность одновременности - это не постулат СТО, а следствие из других постулатов. Два, мысленным экспериментом его опровергнуть невозможно (по внутренней непротиворечивости СТО, как и геометрии Евклида). Три, СТО уже давным давно подтверждена, так что если какой-то мысленный эксперимент её не подтверждает, значит он не сможет воплотиться в реальности.
Поздравляю, Шарик. ты балбес. опыт не может подтверждать теорию в принципе. но увы, совковому школоло, никогда не изучавшему хотя бы логики, не понять сего//ckotinko
Опыт (не мысленный) подтверждать теорию может (так же, как и опровергать). А мысленный... имелось в виду, что мысленные результаты мысленного опыта могут не соответствовать реальности, значит он идёт боком.
Опыт (не мысленный) теорию может опровергать и не опровергать теорию. логическая операция импликации ибо.
Обычно, когда ставят опыт, который может выдать новые (ещё не выданные в тысячах других опытах до) результаты, и эти результаты заранее предсказаны теорией, и, что интереснее всего, результаты с теоретическими совпадают (в пределах погрешности), то происходит радостное заявление о подтверждении теории опытом. И чем чаще и больше подтверждений, тем больше доверия к теории. Пока не опровергнут достигнут границ применимости.
Обычно не означает правильно. Да будет вам известно, что может существовать аж N эквивалентных теорий, описывающих реальность одинаково успешно. Любой, кто ботал ОЦОСы, знает о такой куйне, как преобразования Лапласа. Это когда мы берем пространство 1 и его изоморфно отображаем на пространство 2, и все наши формулы переколбашиваются. И везде всё работает, планеты крутятся, и птицы летают. Но только одно из этих пространств настоящее. До кучи, самих преобразований может быть туева хуча. Если я, к примеру, достану преобразования Tangherlini, и сравню их с преобразованиями Лоренца с точки зрения существования парадокса близнецов, Лоренц окажется проигравшим, потому что у Tangherlini парадокса нет в принципе, никогда и нигде. Но общепринят последний, т.к. к нему подогнали слишком много уравнений. Но даже преобразования Лоренца у самого Лоренца были фиктивными - они переводили не в другую СО, а в фиктивное пространство Лапласа, соответствующее этой СО.
Ну и чего ваши опыты могут доказывать, когда речь идёт о преобразованиях, используемых при переходе от одной СО к другой? //ckotinko
> Лоренц окажется проигравшим, потому что у Tangherlini парадокса нет в принципе, никогда и нигде.
А у преобразований Лоренца разве есть какие-то парадоксы?
Ну да, не спорю, альтернативных теорий может быть много, но СТО от этого ложной не становится, так как практически все её аспекты были подтверждены опытом. Опыт этот может служить подтверждением и других теорий. СТО же простая и рабочая теория, что ещё надо (у механики Ньютона есть несколько разных эквивалентных формулировок, не выбрасывать же её) ? А то какими были преобразования Лоренца у самого Лоренца вообще пофиг. Главное то, что они делают в СТО.
в СТО-то как раз они и делают парадокс близнецов. //ckotinko
Почему парадокс Близнецов назван парадоксом Близнецов? Потому же, что и парадокс Монти Холла назван парадоксом Монти Холла: потому что он противоречит здравому смыслу обывателя. Сам по себе парадокс Близнецов не содержит никаких внутренних противоречий и, строго говоря, не парадокс. Точнее, если верить сраной вики: "1) ситуация (высказывание, утверждение, суждение или вывод), которая может существовать в реальности, но не имеет логического объяснения. 2) В науке формально-логическое противоречие, которое за некоторых условий возникает в процессе логического мышления (доведения). 3) Мысль, мнение, что поразительно расходится с общепризнанными взглядами, (устоявшимися теориями), и может быть не ошибочной, а реально отображать факты с неожиданной стороны (точки зрения)". Он не является парадоксом типа 1 и 2, но является парадоксом типа 3.
Пиздец и ужос. Вот из таких индоктринированных дебилов, которые даже надписи на русском языке понимают альтернативным способом, и получаются защитники теории относительности. Можно, конечно, было спросить, мол схуяли "Он не является парадоксом типа 1 и 2, но является парадоксом типа 3", но, походу это бессмысленная трата времени. //ckotinko
А ты научись по-русски читать для начала. А для не понимающих определений повторяю: "парадокс Близнецов" противоречит здравому смыслу обывателя и может называться "парадоксом" только в этом смысле. "Парадокс Близнецов" не является логически противоречивым и в этом смысле парадоксом называться не может. Теперь понятно?
сам научись: "2) В науке формально-логическое противоречие, которое за некоторых условий возникает в процессе логического мышления (доведения)". Хотя, конечно, может быть я чего-то не понимаю, и у релятивистов не наука, а обывательский трёп.//ckotinko
Ну. Вот таким "парадоксом" (формально-логическое противоречие) он не является. Парадокс Монти-Холла тоже не является парадоксом в этом смысле.
ну так и пишите сразу, что к науке ваш трёп не имеет никакого отношения//ckotinko
"Парадокс" Близнецов можно не относить к науке (потому что там никаких парадоксов нет). А можно и относить, но с оговоркой: слово "парадокс" в названии не тащит за собой того смысла, который хотят затащить антирелятивисты. Это просто реальный и многократно подтверждённый эффект, который у неподготовленных слушателей вызывает ощущение внутреннего противоречия (которое у них вообще много чего вызывает).
ckotinko, если ты и эти предложения не поймёшь, то вынужден констатировать, что для тебя сложные логические конструкции непостижимы, если ты в этих умудряешься путаться и пытаешься выдумать собственные.
Последний раз говорю: если теория относительности претендует на звание науки, то возникающие в ней парадоксы попадают под пункт, начинающийся со слов "В науке". Оные парадоксы имеют свойство сниматься введением доп. гипотез или модификацией имеющихся, а не переквалификацией парадоксов в кухонные со словами "это просто всем кажется". За сим я откланяюсь, ибо логику и правила русского языка каждому встречному школоло я бесплатно преподавать не собираюсь.//ckotinko
В таком случае вынужден заметить, что "в науке" никакого парадокса Близнецов на самом деле нет, а есть он лишь в сознании обывателей и лжеучёных. И если кто-нибудь считает иначе, то пускай покажет, где именно находится логическое противоречие в парадоксе Близнецов.
Я не понял, что ананимусу не очевидно, что два близница даолжны выгладить как два близнеца, а не как дед с внучком? Парадокс он именно в этом.
Только из-за того, что они называются близнецами? Положи одного в криокамеру и подожди - получишь примерно тот же "парадокс".
Покажи мне хотя бы одну работающую криокамеру! Да и вообще - стареть не только люди могут. Возми, например, бревно, распили его пополам, отправь один в соседнюю голактику и обратно, а одно оставь здесь, на Земле мариноваться в извести с пару милионов лет. Что покажет радиуглеродный анализ (всякие космические излучния отбрасываем)? Правильно! Парадокс? А как же!
Ну это я так, для примера, гипотетическая криокамера, неважно. Я не могу понять - где тут парадокс. Один процесс прошёл за более быстрое время, чем другой. Ну так они и в неравноправных условиях находились.
Разберём, чё ты там накатал.
>Есть прямая а. На ней стоит ряд будильников.
>Есть прямая б, она пересекает прямую а
>точка пересечения находит в стороне от ряда будильников
Вот последнее не понял, будильников на всю прямую не хватило?
>Прямая б движется равномерно параллельно или перпендикулярно прямой б
Что? Ещё и перпендикулярно? Тяжело было вспомнить термин "параллельный перенос"?
Ну ладно, пронёс ты свою хуйню, будильники теперь синхронизированы. И чё дальше? Где опровержение-то?
Второе. Опять настроил будильники. И чё?
А посчитать влом? Там получется разница между часами отсинхронизоваными светом, причём разница - линейно растёт с расстоянием. Т.е. способ синхронизации "механическй" и "световой" дают разные результаты в предположении, что СТО верно (что очевидно, надеюсь). Не парадокс ли?
Влом. Показывай свои расчёты.
Подробно влом. Распрями полёт фанразии и представь себе летящий равномерно и прямолинейно относительно ИСО по имени OX (читать "оу-икс") будильник. Пусть его скорость V. Пусть это хитрый будильник. Пусть он выставляет стрелочки любого другого будильника мимо которого он пролетает в такое же положение как и у него в момент пролёта. Дополнительно попросим его быть таким, что бы в нуле (в т. О - читать "оу"), он выставил покоящийся будильник в ноль. Для простоты. Тогда если некто захочит отсинхронизировать часы в OX по Эйнштейну после пролёта хитрого будильника, то он обнаружит, что время на будильниках будет отличатся на Kx, где x - координата x синхронизуемого будильника, и K - коофициент равный (1-sqrt(1-V^2/c^2))/V (выводиться елементарно, однака). Самое забавное, что при такой "хитрой" синхронизации исчезают все "релятивисткие" эффекты - и сокращение ТЕМПА хода часов (в СТО такого ведь нет?) и сокращение длинны. Зато свет бегает как бог на душу положит.
При чём "синхронизация" зависит от скорости будильника. Если разные будильники провести с разными скоростями - то и часы "синхронизируются" по разному. И если что-то там стало незаметно на одной скорости, то не проявятся ли глюки на другой?

Мнимые временные единицы

Кто-нибудь разъяснит тупому непонятливому гуманитарию, что означает данное понятие?

Мнимые - значит выскакивает корень из минус единицы. Короче типа как деления на ноль - абсурд.
Активно используются в квантмехе, тащемта.

Мир Минковского

Как считает Анонимус, стоит ли пилить подстатью про Мир Минковского, в которой кратко, с лёгким налёта матана и несколькими диаграммками, объяснить всю суть СТО?

Нет, нет. Думаю нет. Во первых - диаграммы Минковского совсем даже и близко не мем и подавляющее большинство посетителей луркмора, вообще знающих про то что такое что-то с фамилией "Минковский" существует - узнало именно из этой статьи. Во вторых - там реально математика с геометрией, напряг мозгов и никакого фана, а луркмор всё-таки ресурс развлекательный и если и проявляет свойства образовательного - то только в той мере чтобы удовлетворить развлекательность. В этом же смысле великолепно написаны статьи и про квантмех и про теорию струн... Так что нет. Думаю нет.
Ну, тут уже давно описывают не мемы. Хотя фольклор, современные культуры-субкультуры, да...

Наркоманы штоле сука?

Да нету никаких ТО! Всё это выдумки жыдов, что сводить народ с ума! После прочтения, появилось чувство тревоги и страха, а также какие-то странные голоса из под моего стола. Updated: А сейчас моя кошка, обещает перерезать мне горло, но я ей не ве

лоренцевы сокращения

Суть, и это написано во многих учебниках и справочниках, состоит в "относительном" изменении длин. И теорию относительности нужно рассматривать через структуру света. т.е. через корпускулярно-волновую теорию света (следствием которой и с тала СТО): Если представить себе дугу бильярдных шаров, катящихся вдоль стола. Путь этой шеренги перпендикулярно перелетает кий. Скорость кия возьмем сопоставимой со скоростью движения шаров. Часть шаров, ударившись о кий покатится обратно. Если скорость кия будет выше скорости шаров, то откатится успеют только первый несколько шаров, вертевшиеся с ним. Если скорость кия будет медленнее (или он просто будет лежать) - отразится столько шаров, сколько необходимо, что бы покрыть длину кия. При этом количество отразившихся шаров и есть "относительное (визуальное) сокращение".

новые плашки

я дефсвительно нихуя не понял, и считаю, что в статье действительно очень много букв и знакв.

а ведь плашечка {{brain}} стоит совсем не зря --следящий за статьёй и откативший твою правку неймфажик

Ошибка в разделе Относительность одновременности. Преобразования Лоренца.

Если кто-то подумает, что из ракеты мы будем при таких пертурбациях выглядеть удлинённо и со спешащими часами — то это верно. С тех пор как ракета испытала такое невзъебенное ускорение - весь остальной мир для неё становится вытянутым и слишком быстрым, как будто "включили перемотку вперед на видеокассете вселенной"

Это совершенно неверно, это противоречит ТО. С точки зрения ракеты, двигающейся со скоростью, сопоставимой со скоростью света относительно неких "нас", мы становимся не растянуты (что, есть какие-то растяжения Лоренца?), а, наоборот, сокращены. То есть, фактически, чем быстрее летит ракета, тем более сокращено (физически короче*) пространство, по которому она летит. С точки зрения ракеты, конечно же. Потому-то оно и относительное!

  • Если для внешнего наблюдателя расстояние от дерева до дерева, мимо которых пролетела ракета будет равно, например, 1 км, то при измерении с ракеты мы получим меньшую величину. Например, 500 метров. И это не глюк, а действительное следствие теории относительности и свойства мира, в котором мы живем. Простраство-время — штука местами относительная.

То же самое и со временем. Для нас на ракете, движущейся с околосветовой скоростью, не просто сжимается пространство, но и вместе с этим сжатием замедляется наружное время. А для окружающего пространства замедляется НАШЕ время, на ракете, и никакого тут противоречия нет. Не надо только делать ту ошибку с введением растяжений Лоренца. Растяжений нет, есть сокращения!

  • Спасибо. Исправил на вариант, который и был в статье изначально. Это кто-то повандалил слиганца.

Я смотрю, релятивисты уже поправили текст про отношения ньютоновой и ихней каличной механики. Да как поправили, с гонором: "мол долбоёбу написавшему сиё сообщаем...". Правда, окромя гонора, у деточки нихуя за душой больше нету, поэтому с первых же слов деточка расписалась в своём долбоебизме:

„Одни приводили аргументы, что, мол, преобразования Галлилея это афинные преобразования, а преобразования Лоренца это гиперболический поворот, и этот самый поворот никак не может в пределе переходить во что-то афинное.“

Для начала, деточке бы следовало узнать определение аффинного преобразования. Глядишь бы и узнала, что преобразования Лоренца сами по себе аффинные. Само слово "аффинный" оказалось настолько загадочным и непонятным для деточки, что последняя наделила его в своём воображении какими-то магическими свойствами, вроде "аффинный-это из ньютона":

„А это значит, что в предельном случае малых скоростей поворот можно считать обычным, а не гиперболическим. А обычный поворот это и есть афинное преобразование, инфа 100%.“

Во-первых, деточка не знает, чем отличается гиперболический поворот от обычного. Вообще. Ни ухом ни рылом. Во-вторых, Скотинки писали не о "чем-то аффинном" вообще, а о весьма конкретном параллельном переносе, коий неэквивалентен ни гиперболическому, ни обычному повороту. Этот тезис деточка опровергать не стала, а вместо этого подменила параллельный перенос на "что-то аффинное". Вот такие тупые деточки пошли в наше время. И ТО деточка защищает, судя по всему, рефлекторно, чтоб сойти за умного. //ckotinko

Безмозглые детки

Я смотрю, релятивисты уже поправили текст про отношения ньютоновой и ихней каличной механики. Да как поправили, с гонором: «мол долбоёбу написавшему сиё сообщаем…». Правда, окромя гонора, у деточки нихуя за душой больше нету, поэтому с первых же слов деточка расписалась в своём долбоебизме:

«Одни приводили аргументы, что, мол, преобразования Галлилея это афинные преобразования, а преобразования Лоренца это гиперболический поворот, и этот самый поворот никак не может в пределе переходить во что-то афинное.»

Для начала, деточке бы следовало узнать определение аффинного преобразования. Глядишь бы и узнала, что преобразования Лоренца сами по себе аффинные. Само слово «аффинный» оказалось настолько загадочным и непонятным для деточки, что последняя наделила его в своём воображении какими-то магическими свойствами, вроде «аффинный-это из ньютона»:

«А это значит, что в предельном случае малых скоростей поворот можно считать обычным, а не гиперболическим. А обычный поворот это и есть афинное преобразование, инфа 100%.»

Во-первых, деточка не знает, чем отличается гиперболический поворот от обычного. Вообще. Ни ухом ни рылом. Во-вторых, Скотинки писали не о «чем-то аффинном» вообще, а о весьма конкретном параллельном переносе, коий неэквивалентен ни гиперболическому, ни обычному повороту. Этот тезис деточка опровергать не стала, а вместо этого подменила параллельный перенос на «что-то аффинное». Вот такие тупые деточки пошли в наше время. И ТО деточка защищает, судя по всему, рефлекторно, чтоб сойти за умного. //ckotinko

  • Скотинко! Рад вас здесь видеть) Впариваете свои полные производные, когда мы с вами уже несколько месяцев назад у вас в жж совместно установили, что уравнения Максвелла остаются Лоренц-инвариантными и с полной производной в законе Фарадея? Или что-то новое придумали? От вас же идут лучи НЕНАВИСТИ! С новым годом! //locutus
    • И вас с 4м марта. Кстати, нихера подобного мы не установили: это возможно только если признать высказывание "ур-я максвелла неинвариантны отн. преобразований максвелла" ложным. В противном случае оказывается что у.м. опровергают сами себя. Факт замены полной производной на частную Хэвисайдом - общеизвестен, вы можете прочитать про него в гугле на куче языков. //ckotinko
  • Спокойствие, только спокойствие. Без всяких гиперболизмов и афин вспоминаем что такое «одна теория есть приближение или наоборот уточнение другой». А значит оная фраза лишь то что в определенных рамках эти теории будут выдавать практически одинаковые (но не обязательно равные, достаточно лишь очень близких значений) результаты. И с этим у классической и релятивиской механик полный порядок — погрешности даже на астрономических расстояниях и километровых скоростях меньше тысячных процента. Так что, ckotinko, негодуешь ты зря и не по делу.
    • а)Утверждение «ньютономеханика не эквивалентна СТО даже при малых скоростях» само по себе никак СТО не угрожает. Точно также как и расходждения между теорией Пуанкаре и ньютономеханикой никак не запрещают процессам у пуанкаре идти так, как будто бы ньютон был прав. б)расхождение никуда не девается и по прежнему имеет место. Оно только увеличивается с ростом расстояний — то есть для любой скорости найдется такое расстояние, что пиздец.
      • С ростом расстояний (и времен, кстати тоже) принципиально ничего не меняется. Относительная погрешность будет сохранятся неизменной. Проще пояснить на том же примере коммунистических белок и стрелок — то что у них там по СТО «сожрётся ажно целых полтора километра (!)» при полёте на марс… да совершенно до пизды будет конструкторам аппарата — это такая мелочь на общей дистанции, там других «помех» гораздо больше, что строить они будут по Ньютону, ИЧСХ, аппарат долетит. И даже если в соседнюю галактику отправят — всё то же самое будет, запасы топлива пересчитывать не понадобится, то что за тысячи/миллионы лет полёта с нерелятивиской скоростью набегут какие то разногласия в дни/годы в показаниях часов — да они просто «потеряются» в погрешностях кварцевого генератора корабельного хронометра, его даже не заметят и т. д. и т. п. Так что Ньютон — вполне себе приближение Эйнштейна и при малых скоростях обнаружить расхождения даже на гигантских масштабах можно только высокоточными приборами.

ни хуя не понял

ни хуя не понял. Не понял я ни хуя

Почитай книжку, учебник. Здесь не просветительский портал. Сходи на физический форум.

Нобелевка Эйнштейна

Забавно, но факт. Эйнштейн получил ноблевку... Но не за СТО, и даже не ОТО, а за описание фотоэффекта. Что по сравнению с этими эпичными трудами составляет эпсилон.

Школьник-кун ты все понял?

Тогда ответь на простые вопросы:

  1. Большой адронный коллайдер выпустил два электрона в противоположные стороны, каждый со скростью 0.999с. С какой скоростью увеличивается расстояние межу ними с точки зрения ебанутого ученого, поставившего этот эксперимент?
  2. Какой скоростью ограничена скорость перемещения солнечного зайчика?
  3. Есть две длинные линейки пересекающиеся под острым углом α. Линейки двигаются со скростью 0,999c, скорость каждой линейки перпендикулярна ей. С какой скростью движется точка пересечения?